1. Наудачу выбрано натуральное число, не превосходящее 20. Какова вероятность того, что это число кратно 5?
Решение
В нашем случае элементарным исходом является выбор одного числа от до
— пространство элементарных исходов.
— количество всех элементарных исходов.
Итак, все элементарные исходы равновозможны, т. к. натуральное число выбирается наудачу и пространство элементарных исходов конечно, следовательно, эксперимент, описанный в задаче, удовлетворяет классическому определению вероятности.
— выбранное число является делителем
— множество благоприятствующих исходов.
— количество всех благоприятствующих исходов.
— искомая вероятность.
Ответ:
2. В урне три белых и пять черных шаров. Наудачу вынимают два шара. Какова вероятность того, что эти шары разных цветов?
Решение
Число N всех равновероятных исходов испытания равно числу способов, которыми можно из 8 шаров вынуть два, т. е. числу сочетаний из 8 элементов по 2:
Число благоприятствующих исходов (числу способов, которыми можно из 3 белых шаров вынуть один и из 5 чёрных шаровы вынуть один):
Следовательно, искомая вероятность
Ответ:
3. Вероятность того, что изготовленная на первом станке деталь будет первосортной, равна 0,7. Для второго станка эта вероятность равна 0,8. На первом станке изготовлены всего 2 детали, на втором – 3. Найти вероятность того, что все детали первосортные.
Решение
Так как изготовление деталей на станках независимые события. То, по теореме умножения вероятностей независимых событий, искомая вероятность равна
Ответ:
4. В тире имеется 5 винтовок, вероятности попадания в цель из которых равны соответственно 0.5, 0.6, 0,7, 0,8 и 0,9. Определить вероятность попадания из взятой наугад винтовки.
Решение
Определим гипотезы:
Hi = {выбрана i-тая винтовка} i = 1, 2, 3, 4, 5.
A={попадание в мишень с одного выстрела}
A|Hi ={попадание в мишень из i-той винтовки}
Тогда их вероятности, по условию,
По формуле полной вероятности
Ответ:
5. 30% изделий некоторого предприятия – продукция высшего сорта. Приобретено 4 изделия этого предприятия. Какова вероятность того, что 2 из них высшего сорта?
Решение
Используем схему Бернулли. Здесь n= 4, m=2, p = 0,3, q =1-0,3= 0,7. Тогда по формуле
Вероятность того, что 2 изделия из 4 будут высшего сорта:
Ответ:
6. Пусть n – число независимых испытаний, р – вероятность появления события А в отдельном испытании, k – число наступлений события А за n испытаний:
А) , . Найти
Б) Найти , если ,
Решение
а) Так как n = 600 – велико, то в данном случае применяем локальную формулу Лапласа:
, где .
По условию задачи , , , .
Вычислим .
По таблице находим .
Теперь вычислим искомую вероятность
.
б) Вероятность того, что отклонение частости события от вероятности при n независимых испытаниях не превышает заданного , находится по формуле
Тогда, по условию
Имеем
Используем таблицу значений функции Лапласа
Отвте: ,
7. Случайная величина Х распределена по закону Коши: . Найьти коэффициент а, функцию распределения , вероятность попадания с. в. Х на интервал . Существует ли для с. в. Х числовые характеристики МХ и ДХ?
Решение
Найдём значение параметра а:
Тогда
Найдём функцию распределения
Найдём вероятность попадания с. в. Х на интервал
Посчитаем математическое ожидание и дисперсию:
Следовательно, для данной случайной величины не существуют ни М(Х) ни Д(Х)
8. Изделия испытываются на надёжность. Вероятность выхода из строя за время испытания для каждого изделия равна 0,9. Испытания заканчиваются после первого же изделия, не выдержавшего испытания. Найти математическое ожидание числа испытаний.
Решение
— число испытаний.
— вероятность того, что изделие выдержало испытание.
Если случайные величины независимы, то их ковариации равна нулю. В нашем случае cov(X, Y) ≠ 0.
Коэффициент корреляции.
,
Ответ:
1. Брошены 3 монеты. Найти вероятность того, что выпадут 2 «герба».
Решение
В нашем случае элементарным исходом является выпадение одной из возможных комбинаций. Обозначим «г»-выпадение герба на монете, «р» — выпадение решки на монете. Тогда возможны ситуации:
Ггг, ргг, грг, ррг, ррр, ргр, ггр, грр
— пространство элементарных исходов.
N=8 — количество всех элементарных исходов.
Итак, все элементарные исходы равновозможны, следовательно эксперимент, описанный в задаче, удовлетворяет классическому определению вероятности.
— выпало 2 герба
— множество благоприятствующих исходов.
— количество всех благоприятствующих исходов.
— искомая вероятность.
Ответ:
2. Из 10 карточек с буквами А, Д, А, Б, А, Д, В, Г, Д, А наудачу выбираются 3. Какова вероятность того, что из этих карточек можно сложить слово «два»?
Решение
Число всевозможных исходов-количество способов вытянуть 3 карточки из 10.
A = {из отобранных карточек можно составить слово «два»}
Число благоприятных исходов-количество способов вытянуть 3 буквы из которых можно составить слово «раз»:
M=3*1*4=12 – по теореме умножения вероятностей независимых событий.
Используем формулу классического определения вероятности:
Ответ:
3. Стрелок стреляет 3 раза по удаляющейся цели. Вероятность попадания при первом выстреле – 0,8, при втором – 0,7 и при третьем – 0,6. Какова вероятность хотя бы одного попадания?
Решение
Вероятность не попасть при первом выстреле 1-0,8=0,2, вероятность не попасть при втором выстреле 1-0,7=0,3, вероятность не попасть при третьем выстреле 1-0,6=0,4.
По теореме об умножении вероятностей независимых событий, вероятность не попасть при трёх выстрелах 0,2*0,3*0,4=0,024.
Значит вероятность хотя бы одного попадания (противоположное событие): 1-0,024=0,976.
Ответ: 0,976.
4. 60% школьников – девочки. 80% девочек и 75% мальчиков имеют билеты в театр. В учительскую принесли потерянный кем-то билет. Какова вероятность того, что этот билет принадлежит девочке?
Решение
Определим гипотезы:
h2 = билет потеряла девочки
h3 = билет потерял мальчик
,
A={билет был потерян}
A|h2 ={был потерян билет, и он принадлежал девочке}
A|h3 ={был потерян билет, и он принадлежал мальчику}
Тогда их вероятности, по условию,
По формуле Байеса
Ответ:
5. Изделия некоторого предприятия содержат 5% брака. Найти вероятность того, что среди 5 взятых наудачу изделий будут 2 бракованных.
Решение
Используем схему Бернулли.
Здесь n=5, m=2, p=0,05, q=1-0,05=0,95. Тогда по формуле
Вероятность того, что 2 изделия из 5 будут бракованные:
Ответ:
6. Пусть n – число независимых испытаний, р – вероятность появления события А в отдельном испытании, k – число наступлений события А за n испытаний:
А) , . Найти
Б) Найти , если ,
Решение
а) Так как n = 600 – велико, то в данном случае применяем интегральную формулу Лапласа:
= Ф(х´´) – Ф(х´), где х´ = ; х´´ = ,
т. к. , , , , имеем
Х´ = ; х´´ = .
По таблицам найдем значения Ф(-1)=-Ф(1)=-0,34134
Тогда искомая вероятность будет
= Ф(1) – Ф(-1) =2Ф(1)= 2*0,34134=0,68268
б) Вероятность того, что отклонение частости события от вероятности при n независимых испытаниях не превышает заданного , находится по формуле
Тогда, по условию
Имеем
Используем таблицу значений функции Лапласа
Ответ: =0,68268,
7. Производятся 20 независимых испытаний, в каждом из которых вероятность появления успеха равна 0,2. Найти дисперсию числа появлений успеха в этих испытаниях.
Решение
Случайная величина X — число появлений успеха при 20 независимых испытаниях. n = 20 — количество независимых испытаний
P = 0.2 — вероятность успеха при одном испытании
Q = 1 — p = 1 — 0.2 = 0.8
Случайная величина X имеет распределение Бернулли с параметрами
N = 20 и p = 0.2
D(X) = npq = 20*(0.2)*(0.8) = 3.2
Ответ: D(X)=3.2
8. Случайная величина Х равномерно распределена на . Построить графики и . Найти , если с. в.
Решение
Так как случайная величина распределена равномерно, то
, то есть
Построим график
Известно, что F(x)=
Поэтому,
Если х<-1, то F(x)==0;
Если -1≤х≤1, то
Если х>1, то F(x)= 1
Таким образом,
Построим график
Как известно, дисперсия равномерно распределённой случайной величины равна . Тогда в нашем случае
Если случайные величины независимы, то их ковариации равна нулю. В нашем случае cov(X, Y) ≠ 0.
Коэффициент корреляции.
,
Ответ:
< Предыдущая
Следующая >
Контрольная по теории вероятностей 1
Ответы к контрольной работе по теории вероятности помогут студентам первых курсов, изучающих математические дисциплины. Задания охватывают много теоретического материала, а обоснование их решения пригодится каждому студенту.
Задача 1. Куб все грани которого закрашены, распилен на 1000 кубиков одинаковых размеров. Определить вероятность того что кубик вытянутый наугад будет иметь:
а) одну закрашеную грань;
б) две закрашеные грани.
Вычисления: Если куб распилить на кубики одинакового размера то все грани будут поделены на 100 квадратов. (Примерно как на рисунке) Дальше по условию кубик должен иметь одну закрашенную грань — это значит что кубики должны принадлежать внешней поверхности но не лежать на ребрах куба (2 закрашеные поверхности) и не на углах — имеют три закрашеные поверхности. Следовательно, искомое количество равно произведению 6 граней на количество кубиков в квадрате размером 8*8. 6*8*8=384 – кубики с 1 закрашеной поверхностью. Вероятность равна количеству благоприятных событий к общему их количеству P=384/1000=0,384. б) Две закрашеные грани имеют кубики по ребрам без самих вершин куба. На одном ребре будет 8 таких кубиков. Всего в кубе 12 ребер, поэтому две закрашенные грани имеют 8*12=96 кубиков. А вероятность вытянуть их среди 1000 всех равная P=96/1000=0,096. На этом задание решено и переходим к следующему.
Задача 2. На одинаковых карточках написаны буквы А, А, А, Н, Н, С. Какова вероятность того, что случайно разместив карточки в ряд, получим слово АНАНАС? Вычисления: Нужно рассуждать всегда от того, что известно. Дано 3 буквы А, 2-Н, и 1 — С, всего их 6. Начнем выбирать буквы для слова «ананас». Первой идет буква А, которую мы можем выбрать 3 способами из 6, потому что есть 3 буквы А среди 6 известных. Поэтому вероятность вытянуть первой А равна P1=3/6=1/2. Вторая буква Н, но не следует забывать, что после того как вытащили А остается 5 букв для выбора. Поэтому вероятность вытянуть под 2 номером Н равна P2=2/5. Следующую А вероятность вытянуть среди 4, что осталось P3=2/4. Далее Н можно извлечь из вероятностью P4=1/3. Чем ближе к концу тем больше вероятность, и уже А можем извлечь при P5=1/2. После этого остается одна карточка С, поэтому вероятность ее вытащить равна 100 процентам или P6=1. Вероятность составить слово АНАНАС равна произведению вероятностей P=3/6*2/5*2/4*1/3*1/2*1=1/60=0,016(6). На этом и базируются подобные задачи по теории вероятностей.
Задача 3. Из партии изделий товаровед наугад выбирает образцы. Вероятность того что наугад взятое изделие окажется высшего сорта равна 0,8. Найти вероятность того, что среди 3 отобранных изделий будет два изделия высшего сорта? Вычисления: Данный пример на применение формулы Бернулли. p=0,8; q=1-0,8=0,2. Вероятность вычисляем по формуле
Если объяснять не на языке формул, то нужно составить комбинации из трех событий, два из которых благоприятны, а одно нет. Это можно записать суммой произведений
Оба варианта являются равносильными, только первый можем применить во всех задачах, а второй в подобных к рассмотреной.
Задача 4. Из пяти стрелков двое попадают в цель с вероятностью 0,6 и трое с вероятностью 0,4. Что вероятнее: наугад выбранный стрелок попадает в цель или нет? Вычисления: По формуле полной вероятности определяем вероятность, что стрелок попадет. P=2/5*0,6+3/5*0,4=0,24+0,24=0,48. Вероятность меньше P<0,5, следовательно вероятнее что наугад выбранный стрелок не попадет в цель. Вероятность не попадания составляет
или P=2/5*(1-0,6)+3/5*(1-0,4)=0,16+0,36=0,52.
Задача 5. C 20 студентов, пришедших на экзамен, 10 подготовлены отлично (знают все вопросы), 7 хорошо (знают по 35 вопросов), а 3 плохо (10 вопросов). В программе 40 вопросов. Наугад вызванный студент ответил на три вопроса билета. Какова вероятность того, что он подготовлен на
а) отлично;
б) плохо.
Вычисления: Суть задачи заключается в том что студент ответил на три вопроса билета, то есть на все что были заданы, а вот какова вероятность их вытянуть мы сейчас вычислим. Найдем вероятность что студент ответил на три вопроса правильно. Это будет отношение количества студентов ко всей группе умноженное на вероятность вытянуть билеты которые они знают среди всех возможных
Теперь найдем вероятность что студент принадлежит группе которая подготовлена «на отлично». Это равносильно доле первого слагаемого предварительной вероятности, к самой вероятности
Вероятность, что студент принадлежит группе которая плохо подготовилась достаточно мала и равна 0,00216.
На этом задание выполнено. Хорошо его разберите и запомните как вычислять, поскольку на контрольных и тестах оно распространено.
Задача 6. Монету бросают 5 раз. Найти вероятность того что герб выпадет менее 3 раз? Вычисления: Вероятность вытянуть герб или решку равносильна и равна 0,5. Менее 3 раз означает, что герб может выпасть либо 0, либо 1, либо 2 раза. «Или» всегда в вероятности в операциях сказывается добавлением. Вероятности находим по формуле Бернулли
Поскольку p=q=0,5, то вероятность равна
Вероятность равна 0,5.
Задача 7. При штамповке металлических клемм получается в среднем 90% стандартных. Найти вероятность того что среди 900 клемм стандартными будут не менее 790 и не более 820 клемм.
Вычисления: Вычисления необходимо проводить по интегральной теореме Муавра-Лапласа. Записываем известные величины n=900; p=0,9; q=1-0,9=0,1; k1=790; k2=820. Вероятность находим по формуле
где интегральная функция Лапласа
Значение x1, x2 вычисляем по формуле
Далее находим вероятность что среди 900 клемм стандартными будут не менее 790 и не более 820
Если объясните подобные задачи так как здесь написано то за контрольную по вероятности получите оценку «отлично».
Вероятность
Как вероятно что-то должно произойти.
Многие события невозможно предсказать с полной уверенностью. Лучшее, что мы можем сказать, это насколько вероятно они должны произойти, используя идею вероятности.
Подбрасывание монеты
При подбрасывании монеты возможны два исхода:
Головки (H) или хвостовики (T)
Также:
вероятность выпадения монеты H равно ½
вероятность выпадения монеты T равна ½
Бросание игральных костей
При бросании одной кости возможны шесть исходов: 1, 2, 3, 4, 5, 6 .
Вероятность любого из них равна 1 6
Вероятность
В целом:
Вероятность события = Количество способов, которыми это может произойти Общее количество исходов
Пример: вероятность выпадения «4» кубиком
Количество возможных вариантов: 1 (есть только 1 грань с «4»)
Общее количество исходов: 6 (всего 6 граней)
Таким образом, вероятность = 1 6
Пример: в мешке 5 шариков: 4 синих и 1 красный. Какова вероятность того, что будет выбран синий шарик?
Количество возможных вариантов: 4 (есть 4 синих)
Общее количество исходов: 5 (всего 5 шариков)
Таким образом, вероятность = 4 5 = 0,8
Линия вероятности
Мы можем показать вероятность на линии вероятности:
Вероятность всегда между 0 и 1
Вероятность — это всего лишь ориентир
Вероятность не говорит нам точно, что произойдет, это всего лишь ориентир
Пример: подбросьте монету 100 раз, сколько выпадет орла?
Вероятность говорит о том, что орёл имеет ½ шанса, поэтому мы можем ожидать 50 орлов .
Но когда мы попытаемся это сделать, мы можем получить 48 или 55 орлов… или вообще что угодно, но в большинстве случаев это будет число около 50.
Узнайте больше в индексе вероятности.
слов
Некоторые слова имеют особое значение в Вероятности:
Эксперимент : повторяемая процедура с набором возможных результатов.
Пример: Бросание игральных костей
Мы можем бросать кости снова и снова, так что это повторяется.
Набор возможных результатов любого одиночного броска: {1, 2, 3, 4, 5, 6}
Результат: Возможный результат.
Пример: «6» — это один из результатов броска игральной кости.
Испытание: Одиночное выполнение эксперимента.
Пример: Я провел эксперимент с подбрасыванием монеты. После 4 попыток я получил следующие результаты:
Результат
Испытание
Испытание
Испытание
Пробная
Головка
✔
✔
✔
Хвост
✔
Три испытания имели исход «Голова», а одно испытание — «Хвост».
Sample Space: все возможные результаты эксперимента.
Пример: выбор карты из колоды
В колоде 52 карты (не считая джокеров)
Таким образом, Пространство выборки — это все 52 возможные карты : {Туз червей, 2 червей и т. д. .. }
Пространство выборки состоит из точек выборки:
Точка выборки: только один из возможных результатов
Пример: Колода карт
5 треф является точкой отсчета
Король Червей является пробной точкой
«Король» не является точкой отбора. Есть 4 короля, то есть 4 различных точек выборки.
Пример: бросание игральных костей
В этой области выборки имеется 6 различных точек выборки.
Событие: одно или более результаты эксперимента
Пример событий:
Событие может иметь только один исход:
Выпадение решки при подбрасывании монеты
Роллинг «5»
Событие может включать более одного исхода:
Выбор «Короля» из колоды карт (любого из 4 Королей)
Выпадение «четного числа» (2, 4 или 6)
Эй, давайте использовать эти слова, чтобы вы привыкли к ним:
Пример: Алекс хочет узнать, сколько раз выпадет «дубль» при бросании двух костей.
Пространство выборки все возможно Результаты (36 точек выборки):
Чтобы они были умными и успешными, их нужно «пощупать».
Подбрасывание монеты, бросание игральной кости и розыгрыш лотереи — все это примеры случайных событий.
События
«Событие» может быть одним или несколькими исходами.
Примеры:
Событие может быть одним исходом:
Выпадение решки при подбрасывании монеты является событием
Выпадение «5» — это событие.
Событие может включать несколько исходов:
Выбор «Короля» из колоды карт (любой из 4 Королей) также событие
Выпадение «четного числа» (2, 4 или 6) является событием
События могут быть:
Независимый (каждое событие не зависит от других событий),
Зависимый (также называемый «условным», когда событие зависит от других событий)
Взаимоисключающий (события не могут происходить одновременно)
Давайте рассмотрим каждый из этих типов.
Независимые события
События могут быть «независимыми», то есть каждое событие равно не влияет на никакими другими событиями.
Это важная идея! Монета не «знает», что раньше она выпадала орлом… каждый бросок монеты — это совершенная изолированная вещь.
Пример: Вы подбрасываете монету три раза, и каждый раз выпадает «орел»… какова вероятность того, что при следующем подбрасывании тоже выпадет «орел»?
Шанс просто 1/2, или 50%, точно так же, как ЛЮБОЙ ДРУГОЙ подбрасывание монеты.
То, что он сделал в прошлом, не повлияет на текущий бросок!
Некоторые люди думают, что «решка уже настала», но на самом деле действительно следующий бросок монеты совершенно не зависит от любых предыдущих бросков.
Высказывание «Выпадение хвоста» или «Еще одна попытка, моя удача неизбежна» называется Заблуждение игрока
Узнайте больше на независимых мероприятиях.
Зависимые события
Но некоторые события могут быть «зависимыми». .. это значит, что на них могут влиять предыдущие события .
Пример: Взять 2 карты из колоды
После взятия одной карты из колоды доступно на карты меньше , поэтому вероятности меняются!
Давайте посмотрим на шансы получить короля.
Для 1-й карты шанс вытянуть короля 4 из 52
Но для 2-й карты:
Если 1-й картой был король, то 2-й картой будет минус шансов быть королем , так как из оставшейся 51 карты только 3 короля.
Если 1-я карта была , а не королем, то 2-я карта немного больше , вероятно, будет королем, так как 4 из 51 оставшейся карты являются королями.
Это потому, что мы удаляем карты из колоды.
Замена: Когда мы кладем каждую карту обратно после ее взятия, шансы не меняются, так как события независимы .
Без замены: шансы изменятся, а событий зависимый .
Вы можете узнать больше в Зависимые события: условная вероятность
Древовидные диаграммы
Когда у нас есть зависимые события, это помогает сделать «древовидную диаграмму»
Пример: футбольный матч
Вы собираетесь играть в футбол и любите быть вратарем, но это зависит от того, кто сегодня является тренером:
с тренером Сэмом вероятность того, что вы станете вратарем, равна 0,5
с тренером Алексом ваша вероятность быть вратарем равна 0,3
Катер шёл 3 ч против течения реки и 2 ч по течению .Какой путь прошёл катер за эти 5 ч ,если собственная скорость катера 18,6 км/ч ,а скорость течения реки 1,3 км/ч? — вопрос №1069917 — Учеба и наука
Ответы
3 часа он шёл со скоростью 18,6-1,3=17,3 и прошёл 17,3*3=51,9 км
2 часа со скоростью 18,6+1,3=19,9, и прошёл 19,9*2=39,8 км
За 5 часов прошёл 51,9+39,8=91,7 км
25. 05.14
Михаил Александров
Читать ответы
Андрей Андреевич
Читать ответы
Eleonora Gabrielyan
Читать ответы
Посмотреть всех экспертов из раздела Учеба и наука > Математика
Похожие вопросы
В фермерском хозяйстве 3/5 всего поля засеяли пшеницей, а 35 % поля — овощами. 2-5х+3…
Решено
помогите решить задачу по математике за 3 класс с условием пожалуйста
математика
Пользуйтесь нашим приложением
73, 74, 75. Задачи на движение по воде
73, 74, 75. Задачи на движение по воде
Это надо знать
В задачах на движение по воде скорость реки считается постоянной и неизменной.
При движении по течению скорость реки прибавляется к собственной скорости плывущего тела, так как скорость реки помогает двигаться телу.
При движении против течения от собственной скорости вычитается скорость реки, так как в этом случае скорость реки мешает движущемуся телу.
Скорость плота считается равной скорости реки. Пример:
Пусть скорость движения лодки 5 км/ч, а скорость течения — 2 км/ч. 1) 5 + 2= 7 (км/ч) — скорость лодки по течению 2) 5 — 2 = 3 (км/ч) — скорость лодки против течения
Пусть наша лодка проплыла 2 часа по течению реки и 3 часа против течения реки. Найдем расстояние, которое проплывет лодка. 3) 7 ∙ 2 = 14 (км) — плыла лодка по течению 4) 3 ∙ 3 = 9 (км) — плыла лодка против течения 5) 14 + 9 = 23 (км) — все расстояние Ответ: 23 км.
Видеоурок
Домашнее задание
К уроку 73 (на 16.12) П. 4.11
№ 1
Скорость катера в стоячей воде (собственная скорость) 12 км/ч, а скорость течения реки 3 км/ч. Определите: 1) скорость катера по течению и против течения реки; 2) путь катера по течению реки за 3 ч; 3) путь катера против течения реки за 5 ч.
№ 2
Скорость катера против течения равна 23 км/ч, а скорость течения 4 км/ч. Найдите скорость катера по течению.
№ 3
Скорость моторной лодки по течению реки равна 14 км/ч/ а скорость течения 3 км/ч. Найдите скорость лодки против течения
К уроку 74 (на 17.12) П. 4.11 № 5.302 Расстояние между пристанями прогулочный теплоход проплывает по течению за 3 ч со скоростью 24 км/ч, а за 4 ч возвращается обратно. Какова скорость катера в стоячей воде и скорость течения реки? № 5.303 Путешественник спустился вниз по течению реки за 2 сут. на плоту, а обратно вернулся теплоходом за 2 ч. Найдите собственную скорость теплохода, если скорость течения реки равна 2 км/ч. № 5.304 Наша Таня громко плачет, уронила в речку мячик. Но проплакав 2 мин, Таня поплыла за мячиком и через 2 мин догнала его. С какой скоростью плыла Таня, если скорость течения реки равна 35 м/мин?
К уроку 75 (на 18. 12) П. 4.11 № 5.306 Вниз по течению реки плывет моторная лодка, а навстречу ей катер. Через какое время после начала движения лодка и катер встретятся, если их собственные скорости равны 12 км/ч и 15 км/ч соответственно, скорость течения реки равна 2 км/ч; лодка и катер начали движение одновременно, находясь на расстоянии 54 км друг от друга?
№ 5.297 Из поселка Веселково одновременно в противоположных направлениях отправились два рейсовых автобуса. Скорость одного автобуса равна 56 км/ч, другого — на 8 км/ч больше. Через сколько часов расстояние между автобусами будет равно 480 км?
Главная страница
Подписаться на:
Сообщения (Atom)
Моторная лодка проплывает 108 км вверх по течению за 3 часа. Обратный путь занимает 2 часа вниз по течению. Какова скорость лодки в стоячей воде и какова скорость куб.
РЕШЕНИЕ: Моторная лодка проходит 108 км против течения за 3 часа. Обратный путь занимает 2 часа вниз по течению. Какова скорость лодки в стоячей воде и какова скорость ку.
Алгебра ->
Настраиваемые средства решения задач Word
-> Путешествия
-> РЕШЕНИЕ: Моторная лодка проплывает 108 км вверх по течению за 3 часа. Обратный путь занимает 2 часа вниз по течению. Какова скорость лодки в стоячей воде и какова скорость ку. Войти
Реклама: более 600 задач по алгебре на edhelper.com
Задачи Word: Путешествие и расстояниеWord
РешателиРешатели
УрокиУроки
Архив ответовОтветы
Нажмите здесь, чтобы увидеть ВСЕ задачи на Travel Word Problems
Вопрос 1095637: Моторная лодка за 3 часа проплывает 108 км вверх по течению. Обратный путь занимает 2 часа вниз по течению. Какова скорость лодки в стоячей воде и какова скорость течения?
Найдено 2 решения от ikleyn, Alan3354 : Ответ от ikleyn(48143) (Показать источник):
Вы можете разместить это решение на ВАШЕМ сайте! .
Пусть u — скорость моторной лодки в стоячей воде, а v — скорость течения.
Эффективная скорость движения вверх по течению равна
= = 36 км/ч.
Это РАЗНИЦА скорости моторной лодки в стоячей воде и скорости течения. Это дает вам ваше первое уравнение
и - v = 36. (1)
Эффективная скорость вниз по течению равна
= = 54 км/ч.
это SUM скорости моторной лодки в стоячей воде и скорости течения. Это дает вам ваше второе уравнение
и + v = 54. (2)
Таким образом, у вас есть эта система двух уравнений с двумя неизвестными.
и - v = 36, (1) и
и + v = 54. (2)
Сложите два уравнения. Ты получишь
2u = 36 + 54 = 90 ====> u = = 45 км/ч.
Итак, вы только что нашли скорость моторной лодки в стоячей воде. Это 45 км/ч.
Тогда из уравнения (2) получаем v = 54 = 45 = 9км/ч - текущая скорость.
Ответить . Скорость катера в стоячей воде 45 км/ч.
Текущая скорость 9 км/ч.
Решено.
———— Это типичная и стандартная проблема с передачей слов туда и обратно в восходящем и нисходящем направлениях. Вы можете найти много похожих полностью решенных задач на восходящие и нисходящие обходы с подробными решениями в уроках. — Проблемы с ветром и течением – Дополнительные проблемы при восходящем и нисходящем обходах — Задачи ветра и течения, решаемые квадратными уравнениями — Плот без двигателя плывет вниз по течению реки. — Избранные задачи из архива на лодке, плывущей вверх по течению и вниз по течению на этом сайте.
Внимательно прочтите их и узнайте, как решить подобные проблемы раз и навсегда.
Кроме того, на этом сайте у вас есть бесплатный онлайн-учебник по АЛГЕБРЕ-I. — АЛГЕБРА-I — ВАШ ОНЛАЙН-УЧЕБНИК.
Упомянутые уроки являются частью этого учебника в разделе « Словесные задачи «, тема « Задачи о путешествии и расстоянии «.
Сохранить ссылку на этот онлайн-учебник вместе с его описанием
Бесплатный онлайн-учебник по АЛГЕБРЕ-I https://www.алгебра.com/алгебра/homework/quadratic/lessons/ALGEBRA-I-YOUR-ONLINE-TEXTBOOK.lesson
в свой архив и используйте его при необходимости.
Ответ от Alan3354(69276) (Показать источник):
Вы можете разместить это решение на ВАШЕМ веб-сайте! Шаг 1, найдите 2 скорости вверх и вниз по течению. Скорость лодки относительно воды равна среднему значению 2. ——- Течение равно разнице между скоростью относительно воды и скоростью относительно земли.
Ставочные задачи (системы уравнений с двумя переменными)
Rate Problems (системы уравнений с двумя переменными)
Проблемы со скоростью
Лодка может пройти 16 миль вверх по реке за 2 часа. Та же лодка может пройти 36 миль вниз по течению за 3 часа. Что
скорость лодки в стоячей воде? Какова скорость течения?
Что мы пытаемся найти в этой задаче? Мы хотим найти две вещи: скорость лодки в
стоячая вода и скорость течения. Каждая из этих вещей будет
быть представлено другой переменной:
B = скорость лодки в стоячей воде C = скорость тока
Поскольку у нас есть две переменные, нам нужно найти систему
из двух уравнений, которые нужно решить.
Как найти два нужных уравнения? Проблемы скорости основаны на отношениях Расстояние
= (Ставка)(Время) .
Для организации работы мы составим карту расстояний,
скорость и время, за которое лодка движется как против течения, так и против течения.
Диаграмма даст нам информацию о расстоянии, скорости и времени, которое
нам нужно написать наши два уравнения.
Вот как выглядит диаграмма до того, как мы поместим любой из
наша информация в нем:
Расстояние
Ставка
Время
вверх по течению
вниз по течению
Давайте посмотрим на слова задачи.
Лодка может пройти 16 миль вверх по реке за 2 часа. Мы поместим 16 в нашу диаграмму для расстояния вверх по течению, и мы поставим 2 в
график времени вверх по течению.
Та же лодка может пройти 36 миль вниз по течению за 3 часа. Мы поместим 36 на нашу диаграмму для расстояния вниз по течению, и мы поставим 3
на графике для времени вниз по течению.
Теперь наша диаграмма выглядит так:
Расстояние
Ставка
Время
вверх по течению
16
2
вниз по течению
36
3
Теперь давайте подумаем о скорости движения лодки. Мы знаем, что если бы лодка стояла на тихом озере, ее мотор двигал бы ее.
со скоростью В миль в час. Но лодка не на тихом озере;
он движется вверх и вниз по течению реки. Если лодка плывет
вверх по течению, течение (которое составляет C миль в час) будет давить на
катер, и скорость катера уменьшится на С миль в час.
Результирующая скорость лодки (движущейся против течения) равна B-C миль в час.
С другой стороны, если лодка движется вниз по течению, течение
толкать лодку быстрее, и скорость лодки увеличится на С миль
в час. Результирующая скорость лодки (плывущей по течению)
составляет B+C миль в час. Мы поместим эту информацию в нашу диаграмму:
В самом названии уже подчеркивается, что это фигура, у которой три угла. Следовательно, в многоугольнике их может быть много, т.е. больше, чем три. Например, изобразим пятиугольник – фигуру с пятью углами.
Многоугольник – фигура, состоящая из нескольких точек (больше двух) и соответствующего количества отрезков, которые их последовательно соединяют. Эти точки называются вершинами многоугольника, а отрезки – сторонами. При этом никакие две смежные стороны не лежат на одной прямой и никакие две несмежные стороны не пересекаются.
Любой многоугольник разделяет плоскость на две области: внутреннюю и внешнюю. Внутреннюю область также относят к многоугольнику.
Иными словами, например, когда говорят о пятиугольнике А1А2А3А4А5, имеют в виду и всю его внутреннюю область, и границу. А ко внутренней области относятся и все точки, которые лежат внутри многоугольника.
Многоугольники еще иногда называют n-угольниками, чтобы подчеркнуть, что рассматривается общий случай наличия какого-то неизвестного количества углов (n штук).
Периметр многоугольника – сумма длин сторон многоугольника.
Отрезок, соединяющий любые две противоположные вершины, называется диагональю многоугольника.
Многоугольники делятся на выпуклые и невыпуклые. Например, многоугольник, изображенный на рисунке выше, является выпуклым, а на рисунке ниже – невыпуклым.
Многоугольник называется выпуклым, если при проведении прямой через любую из его сторон весь многоугольник лежит только по одну сторону от этой прямой. Невыпуклыми являются все остальные многоугольники.
Правильный многоугольник – это выпуклый многоугольник, у которого все стороны и углы равны.
Существенное отличие четырехугольника от треугольника в том, что он может быть выпуклым или невыпуклым.
Очень важное различие, о котором знает каждый плотник, состоит в том, что треугольник – «жесткая» фигура, а четырехугольник (как и все остальные многоугольники) – «нежесткая».
У треугольника невозможно изменить его форму, не изменив длин сторон. При этом у любого четырехугольника можно изменить его форму, не меняя длины сторон. На практике это будет означать, что треугольник, сколоченный из трех дощечек, будет жестким, не будет сминаться даже при сильных воздействиях, а четырехугольник при достаточной нагрузке со стороны изменит свою форму.
Для описания свойств многоугольников существуют две важнейшие теоремы об их углах: теорема о сумме внутренних углов выпуклого многоугольникаитеорема о сумме внешних углов выпуклого многоугольника.
Теорема. О сумме внутренних углов выпуклого многоугольника (n-угольника).
Сумма углов n-угольника равна 180°·(n-2).
Математическая запись: ∠A1+∠A2+…+∠An=180°(n-2), где n– количество его углов (сторон).
Вспомним, что любой четырехугольник состоит из двух треугольников (достаточно провести диагональ). Но сумма углов каждого из них одинакова и равна 1800, значит, сумма углов четырехугольника 3600.
Теорема. О сумме внешних углов выпуклого многоугольника (n-угольника).
∠1’+∠2’+…+∠n’=360°, где n – количество его углов (сторон), а ∠1′,…,∠n’ – внешние углы, по одному от каждой вершины.
Подготовка школьников к ЕГЭ и ОГЭ (Справочник по математике — Планиметрия
Поиск по сайту:
Справочник по математике
Геометрия (Планиметрия)
Четырехугольники
Типы четырехугольников
Типы параллелограмов
Типы трапеций
Типы четырёхугольников
Классификация треугольников изложена в разделе нашего справочника «Типы треугольников».
Целью данного раздела является классификация четырёхугольников.
Классификация четырёхугольников по типам представлена на схеме 1.
Схема 1
Рисунки и определения фигур, представленных на схеме 1, даны в следующей таблице.
Фигура
Рисунок
Определение
Четырёхугольник
Четырёхугольник – это часть плоскости, ограниченная замкнутой ломаной линией с четырьмя звеньями без самопересечений.
Выпуклый четырёхугольник
Выпуклый четырёхугольник – это четырёхугольник, который вместе с любыми двумя точками содержит и весь отрезок с концами в этих точках
Невыпуклый четырёхугольник
Четырёхугольник называют невыпуклым, если он не является выпуклым.
Параллелограмм
Параллелограмм – это четырёхугольник, у которого противолежащие стороны параллельны
Трапеция
Трапеция – это четырёхугольник, у которого две стороны параллельны (основания), а две другие стороны не параллельны (боковые стороны).
Дельтоид
Дельтоид – это выпуклый четырёхугольник, состоящий из двух различных равнобедренных треугольников с общим основанием, вершины которых лежат по разные стороны от этого основания.
Четырёхугольник
Четырёхугольник – это часть плоскости, ограниченная замкнутой ломаной линией с четырьмя звеньями без самопересечений.
Выпуклый четырёхугольник
Выпуклый четырёхугольник – это четырёхугольник, который вместе с любыми двумя точками содержит и весь отрезок с концами в этих точках
Невыпуклый четырёхугольник
Четырёхугольник называют невыпуклым, если он не является выпуклым.
Параллелограмм
Параллелограмм – это четырёхугольник, у которого противолежащие стороны параллельны
Трапеция
Трапеция – это четырёхугольник, у которого две стороны параллельны (основания), а две другие стороны не параллельны (боковые стороны).
Дельтоид
Дельтоид – это выпуклый четырёхугольник, состоящий из двух различных равнобедренных треугольников с общим основанием, вершины которых лежат по разные стороны от этого основания.
Типы параллелограммов
На схеме 2 представлена классификация параллелограммов.
Схема 2
Рисунки и определения фигур, представленных на схеме 2, даны в следующей таблице.
Фигура
Рисунок
Определение
Прямоугольник
Прямоугольник – это параллелограмм, у которого все углы прямые.
Ромб
Ромб – это параллелограмм, у которого все стороны равны.
Квадрат
Квадрат – это параллелограмм, у которого все углы прямые и все стороны равны.
Параллелограмм общего вида
Параллелограмм – это четырёхугольник, у которого противолежащие стороны параллельны
Прямоугольник
Прямоугольник – это параллелограмм, у которого все углы прямые.
Ромб
Ромб – это параллелограмм, у которого все стороны равны.
Квадрат
Квадрат – это параллелограмм, у которого все углы прямые и все стороны равны.
Параллелограмм общего вида
Параллелограмм – это четырёхугольник, у которого противолежащие стороны параллельны
Типы трапеций
На схеме 3 представлена классификация трапеций.
Схема 3
Рисунки и определения фигур, представленных на схеме 3, даны в следующей таблице.
Фигура
Рисунок
Определение
Равнобедренная (равнобочная) трапеция
Равнобедренной называют трапецию, у которой боковые стороны равны.
Прямоугольная трапеция
Прямоугольной называют трапецию, у которой одна из боковых сторон перпендикулярна основаниям.
Трапеция общего вида
Трапеция – это четырёхугольник, у которого две стороны параллельны, а две другие стороны не параллельны.
Равнобедренная (равнобочная) трапеция
Равнобедренной называют трапецию, у которой боковые стороны равны.
Прямоугольная трапеция
Прямоугольной называют трапецию, у которой одна из боковых сторон перпендикулярна основаниям.
Трапеция общего вида
Трапеция – это четырёхугольник, у которого две стороны параллельны, а две другие стороны не параллельны.
На сайте можно также ознакомиться с нашими учебными материалами для подготовки к ЕГЭ и ОГЭ по математике.
До ЕГЭ по математике осталось
дней
часов
минут
секунд
НАШИ ПАРТНЕРЫ
«НПО Астек»
«Fastvideo»
Бюро переводов «Медтран»
Независимый бизнес-консультант Е.Самаров
Q5 Объясните, почему прямоугольник является выпуклым четырехугольником…
Перейти к
Упражнение 3.1
Упражнение 3. 2
Упражнение 3.3
Упражнение 3.4
Рациональное число
Линейные уравнения с одной переменной
Понимание четырехугольников
Практическая геометрия
Обработка данных
Квадраты и квадратные корни
Кубы и кубические корни
Сравнение количеств
Алгебраические выражения и тождества
Визуализация твердых фигур
Измерение
Показатели и силы
Прямые и обратные пропорции
Факторизация
Введение в графики
Игра с числами
Главная >
Решения НЦЭРТ
Класс 8
Математика
>
Глава 3. Понимание четырехугольников
>
Упражнение 3.4
>
Вопрос 16
Вопрос 16 Упражнение 3.4
В5) Объясните, почему прямоугольник является выпуклым четырехугольником.
Ответ:
Решение:
Прямоугольник является выпуклым четырехугольником, так как его вершина приподнята и обе его диагонали лежат внутри него.
Связанные вопросы
**Укажите, верно это или нет:** **Все ромбы — воздушные змеи.**
**Укажите, верно это или нет:** **Все параллелограммы являются трапециями.**
**Укажите, верно это или нет:** **Все прямоугольники являются квадратами.**
**Укажите, верно это или нет:** **Все ромбы являются параллелограммами.**
**Укажите, верно это или нет:** **Все квадраты не являются параллелограммами. **
**Укажите, верно это или нет:** **Все воздушные змеи представляют собой ромбы.**
Фейсбук
WhatsApp
Копировать ссылку
Было ли это полезно?
Упражнения
Упражнение 3.1
Упражнение 3.2
Упражнение 3.3
Упражнение 3.4
Главы
Рациональные числа
Линейные уравнения в одной вариабельной
СПАСИДЕНИЯ
ПРАКТИЧЕСКАЯ ГЕЙОМЕТРИЯ
Data Handling Handling Handladlals
. Корни
Кубы и кубические корни
Сравнение величин
Алгебраические выражения и идентичности
Визуализация твердых форм
Mensuration
Экспоненты и мощности
Прямые и обратные пропорции
Фактор
Введение в графики
играют с номерами
. n — 1
$
прямоугольники на каждом шаге моего алгоритма (см. рисунки ниже). Я утверждаю, что $A_{x_n} \ge a_n$ (и $a_n$ также описывается ниже и сходится к $1$ с разумной скоростью).
Пусть $\mathrm{diam}(O)$ — диаметр $O$, и пусть две точки $x$ и $y$ на $\partial O$ такие, что $\| х — у \| = \mathrm{диам}(O)$. (Функция, которая берет две точки в $O$ и выводит их расстояние, непрерывна, а $\partial O$ компактна, так что у нас действительно есть такая пара $(x,y)$.) Это означает, что в точке $x $ касательная, параллельная $\partial O$, проходящая через $x$, ортогональна прямой, проходящей из $x$ в $y$. См. рисунок ниже.
Теперь, поскольку $x$ и $y$ выбраны так, чтобы расстояние между ними было максимальным, векторы, касающиеся $\partial O$ в точках $x$ и $y$, в обоих направлениях «вверх» и «вниз» (см. рисунки) будут указывать внутрь (поскольку они имеют максимальное расстояние), так что «высота» точек на $\partial O$ относительно оси $x-y$ будет вогнутой функцией (минус выпуклая функция) , поэтому имеет максимум. Выберите точку $z_1$ с максимальной высотой для верхней зоны и $z_2$ для нижней зоны. Я сделал еще один рисунок, который рассматривает нижний корпус, а верхний корпус симметричен.
На рисунке я определяю $f(t)$ как площадь определенного прямоугольника. Я определяю $f : (0,a) \to \mathbb R$, полагая $t$ расстоянием от пересечения между осью $x-y$ и точкой, опущенной ортогонально из $z_2$ (которая является нижней точкой) и некоторая точка слева от этой точки на оси $x-y$. Чтобы нарисовать прямоугольник, вы опускаетесь вертикально вниз, затем поворачиваетесь ортогонально, пока не достигнете границы, а затем поворачиваетесь ортогонально и снова останавливаетесь на оси $x-y$. Я определяю прямоугольник таким образом, потому что наихудшее выпуклое множество, имеющее $x$, $y$ и $z$ в этих местах, — это просто два треугольника, следовательно, площадь будет минимальной, и я получу нижнюю границу.
Используя геометрию, вы можете легко увидеть, что
$$
f(t) = (t+\beta)(h-\alpha) = \left(t+\frac{t(d-a)}a \right)\left(h — \frac{ht}a \right) = \frac {dh}a \left( t \left( 1 — \frac ta \right) \right). $$
что дает максимум при $t = \frac a2$ (естественно, не так ли) и максимум при $\frac{3}{16} dh.
Теперь площадь обоих прямоугольников равна $\frac{3}{16}dh_1 +\frac{3}{16}dh_2 = \frac{3}{16}(d(h_1+h_2)) \ge \frac {3}{16}$, а неравенство возникает из-за того, что выпуклое множество можно заключить в прямоугольник шириной $d$ и высотой $h_1+h_2$, где $h_1$ – максимальная высота одной из конструкций, а $h_2$ — другой.
На шаге $1$ я удалил как минимум $r=3/16$ из множества $O$ с 2 прямоугольниками. Оставшееся пространство (представьте себе круг с вписанным в него квадратом, осталось $4$ зон… это новые выпуклые множества, которые я перебираю). Теперь из оставшейся области $(1-r)$ я могу удалить как минимум $3/16$, следовательно, теперь удалена область $r + (1-r)r$. Итерируя таким образом, я получаю
$$
a_1 = r, \qquad a_n = a_{n-1} + (1-a_{n-1})r
$$
а так как эта последовательность явно возрастает и ограничена сверху, то она сходится, и, допуская предел $a = a + (1-a)r$, получаем $a = 1$.
Портфолио проекта «Графики квадратичной функции в зависимости от чисел a, b и c»
Содержание
Портфолио проекта «Графики квадратичной функции в зависимости от чисел a, b и c»
Тема проекта
Предмет, класс
Краткая аннотация проекта
Вопросы, направляющие проект
Основополагающий вопрос
Проблемные вопросы:
План проведения проекта
Визитная карточка проекта
Дидактические цели и ожидаемые результаты обучения
Презентация учителя для выявления представлений и интересов учащихся
Пример продукта проектной деятельности учащихся
Материалы по формирующему и итоговому оцениванию
Описание методов оценивания
Ученик, для которого язык изучения не является родным
Полезные ресурсы
Используемые ресурсы
Бондарчук Светлана Николаевна и Богословская Александра Михайловна
Графики квадратичной функции в зависимости от чисел a, b и c
алгебра, 8 класс.
Данный проект ориентирован на учеников 8 класса базового уровня. Выбранная тема представлена в образовательном стандарте в разделе Алгебра «Числовые функции». В рамках данной темы рассматриваются такие вопросы как:
От чего зависит расположение графика квадратичной функции?
1)от чего зависит направления ветвей параболы?
2)от чего зависит форма параболы?
3)от чего зависит место нахождения вершины параболы?
1. Подготовительный этап (определение целей и задач проекта; составление общего плана проекта).
2. Начало проекта (актуализация знаний учащихся с помощью проблемной ситуации; разбиение учеников на группы и распределение вопросов и тем проекта; обсуждение возможных источников информации).
3. Развитие проекта (поиск и подбор материалов; выдвижение гипотез; проведение исследования; оформление результата исследования в виде обобщенной презентации и решения проблемной задачи; самооценка вклада каждого ученика в коллективную работу).
4. Заключительный этап (защита проделанной работы на итоговой конференции;итоговая рефлексия).
Схема проведения проекта
Обучающие цели
*Изучить расположение графиков квадратичной функции в зависимости от чисел а, b и с;
Воспитательные цели
Формирование навыков коллективной и самостоятельной работы.
Расширение кругозора учащихся.
Развитие умений сравнивать, обобщать, анализировать.
Развитие умения правильно обобщить данные и сделать вывод.
Развивающие цели:
развитие умения работать с Интернет-ресурсами;
развитие умения строить графики квадратичной функции ;
развитие познавательного интереса;
развитие умения работать с большим количеством информации по рассматриваемой проблеме.
Ожидаемые результаты обучения:
По окончанию проекта ученики смогут:
самостоятельно решать жизненного характера с помощью графика квадратичной функции;
ориентироваться в большом количестве разнообразной информации по рассматриваемой проблеме;
критически оценивать свои возможности;
оформлять компьютерные презентации по теме проекта.
123.odp
исследовательская_работа.odp
критерии_оценивания_wiki-статьи.odt
критерии_оценивания_итоговой_презентации.odt
оценка_работы_группы.odt
самооценка_ученика.odt
тест онлайн: http://madam-fonova.ucoz.ru/publ/testy_po_algebre_8_klass/test_3/test_3_variant_2_8_klass/29-1-0-42
тестирование: расставьте знаки >,< или = в исходя из предложенных графиков квадратичной функции
А
парабола, которая раскрывается, имеет самую низкую точку, а парабола, которая раскрывается
вниз имеет наивысшую точку. Самая высокая или самая низкая точка параболы
называется вершиной . Парабола симметрична относительно
вертикальная линия, проходящая через его вершину, называемая осью симметрии .
На рисунке ниже показана раскрывающаяся парабола с вершиной (0,75, 0,875). и ось симметрии x = 0,75.
Вершина
Форма квадратичной функции
Для нахождения вершины параболы напишем функцию
по форме
. В качестве примера рассмотрим функцию . Сначала заполняем квадрат с правой стороны:
Уведомление
что для всех значений x.
Таким образом, f(x) = 2(x — 2) 2 — 1 -1 для всех значений
x и минимальное значение функции равно -1, когда x = 2. Точка (2, -1) является самой нижней точкой на графике, поэтому она является вершиной
параболы. Вертикальная линия x = 2 является осью симметрии.
См. график ниже.
В общем случае называется вершинной формой квадратичного
функция. Когда квадратичная функция записывается в вершинной форме,
мы можем легко определить вершину (h, k). Если коэффициент a > 0 , то парабола открывается вверх и вершина
самая нижняя точка параболы. Мы говорим, что к это минимум
значение квадратичной функции. С другой стороны, если коэффициент a < 0 , то парабола открывается вниз и вершина
высшая точка параболы. В этом случае к является максимальным
значение квадратичной функции. Изучите роль каждого коэффициента
в следующем интерактивном примере.
Пример
Запись
квадратичная функция в вершинной форме. Определить вершину и максимум
или минимальное значение функции.
Решение
Заполним квадрат до
запишите функцию в виде вершины:
Форма вершины , поэтому вершина (3,
-11) . Так как а < 0, парабола открывается вниз и
вершина – самая высокая точка. Функция имеет максимальное значение 11.
Его график показан ниже.
Зная вершину параболы, мы можем определить диапазон квадратичной функции. Рассмотрим функцию . Ранее мы определили, что парабола
имеет минимальное значение -1, возникающее при x = 2. Таким образом, диапазон
квадратичная функция равна {y y -1}. В качестве другого примера, давайте
вернуться к функции в приведенном выше примере. График этой функции
— парабола, развернутая вниз, и максимальное значение функции
равно 11. Следовательно, диапазон квадратичной функции равен
Нахождение
вершина Алгебраически
вершина квадратичной функции также может быть определена алгебраически. Мы первые
предположим, что квадратичная функция имеет две точки пересечения по оси x. Затем
график представляет собой параболу, пересекающую ось x в двух различных точках.
Так как парабола симметрична относительно вертикали, проходящей через
его вершина (ось симметрии) x-координата вершины равна
всегда на полпути между двумя x-перехватами. По квадратичной формуле
два x-перехвата равны
Обратите внимание, что
одно и то же число прибавляется и вычитается из . Отсюда следует, что число находится посередине между Это означает, что x-координата вершины
это . Затем мы можем найти y-координату вершины
путем вычисления Хотя мы предполагали, что квадратичная функция
имел два x-пересечения, когда мы выводили нашу вершинную формулу, она также верна
в двух других случаях, когда парабола имеет одну точку пересечения с абсциссой или вообще не имеет ее.
Пример
Найти
вершина квадратичной функции . Используйте вершину, чтобы определить максимум или минимум
значение функции и найти ее диапазон.
Решение
Вершина
формула дает Чтобы найти вторую координату вершины,
мы оцениваем Вершина параболы (3, 53). С года < 0 , парабола открывается вниз, а вершина является самой высокой
точка. Это дает максимальное значение 53 и диапазон функции Ниже показан график функции.
Вызов
функция , описывающая высоту в
футов мяча через t секунд после того, как он был брошен вверх с вершины
здание высотой 200 футов. Теперь мы можем определить, когда мяч попадает
земли и максимальной высоты, на которую он достигает, а также
время, когда он достигает этой максимальной высоты. Когда мяч коснется земли,
его высота над землей будет равна нулю. Это дает квадратное уравнение . Используя квадратичную формулу, находим, что
решения:
и
(округлено до
два десятичных знака). Поскольку время не может быть отрицательным, мы видим, что
мяч ударяется о землю через 5,21 секунды. Максимальная высота
шара будет задан второй координатой вершины и
время будет первой координатой. Используя вершинную формулу, мы
найти это (округлено до двух знаков после запятой). Следующий
мы оцениваем Это означает, что мяч достигает своего максимума
высота 231,64 фута через 1,41 секунды.
Как найти уравнение параболы с двумя точками и осью симметрии, но без вершины?
Предварительный расчет
Дэвид Дж.
спросил 03.04.19
У меня есть то, что он проходит через 1,4 и 2,7 и что у него есть ось симметрии в точке х = 0, но больше ничего
Подписаться
І
1
Подробнее
Отчет
2 ответа от опытных наставников
Лучший
Новейшие
Самый старый
Автор:
Лучшие новыеСамые старые
Морис Г.
ответил 04.04.19
Репетитор
5,0
(145)
Инженер по образованию, учитель наизусть
Об этом репетиторе ›
Об этом репетиторе ›
Используя форму вершины параболы f(x) = a (x — h ) 2 + k где (h,k) — вершина параболы
Ось симметрии равна x = 0, поэтому h также равно 0
. Подставим каждую точку параболы в форму вершины:
4 = a(1 — 0) 2 + k
4 = а(1) + к
4 = а + к
7 = а(2 — 0) 2 + к
7 = а(4) + к
7 = 4а + к
Нам известна линейная система:
4 = a + k
7 = 4a + k
Вычитание двух уравнений дает нам:
-3 = -3a
a = 1
0
7 90 значение в первое уравнение линейной системы:
4 = 1 + k
k = 3
f(x) = (x — 0) 2 + 3
f( 1 ) = 4 = ( 1 — 0) 2 + 91 f( 3
7 2 ) = 7 = ( 2 — 0) 2 + 3 = 4 + 3
Уравнение параболы через заданные точки и ось симметрии:
f(219) = 90 (х — 0) 2 + 3 = х 2 + 3
Голосовать за 1 Понизить голос
Подробнее
Отчет
Марк М. ответил 03.04.19
Репетитор
5,0
(270)
Учитель математики — высококвалифицированный специалист NCLB
Об этом репетиторе ›
Об этом репетиторе ›
4 = a(x — 1) + k
7 = a(x — 2) + k
Найти a, т. е. исключить k
Подставить любую точку и в любое уравнение со значением «a» и решить для к.
Голосовать за 1 Понизить голос
Подробнее
Отчет
Все еще ищете помощи? Получите правильный ответ, быстро.
Задайте вопрос бесплатно
Получите бесплатный ответ на быстрый вопрос. Ответы на большинство вопросов в течение 4 часов.
Функция — округление x в меньшую сторону (пример floor(4.5)==4.0)
ceiling(x)
Функция — округление x в большую сторону (пример ceiling(4.5)==5.0)
sign(x)
Функция — Знак x
erf(x)
Функция ошибок (или интеграл вероятности)
laplace(x)
Функция Лапласа
asech(x)
Функция — гиперболический арксеканс от x
csch(x)
Функция — гиперболический косеканс от x
sech(x)
Функция — гиперболический секанс от x
acsch(x)
Функция — гиперболический арккосеканс от x
Постоянные:
pi
Число «Пи», которое примерно равно ~3. 2
Функция — Квадрат x
ctg(x)
Функция — Котангенс от x
arcctg(x)
Функция — Арккотангенс от x
arcctgh(x)
Функция — Гиперболический арккотангенс от x
tg(x)
Функция — Тангенс от x
tgh(x)
Функция — Тангенс гиперболический от x
cbrt(x)
Функция — кубический корень из x
gamma(x)
Гамма-функция
LambertW(x)
Функция Ламберта
x! или factorial(x)
Факториал от x
DiracDelta(x)
Дельта-функция Дирака
Heaviside(x)
Функция Хевисайда
Интегральные функции:
Si(x)
Интегральный синус от x
Ci(x)
Интегральный косинус от x
Shi(x)
Интегральный гиперболический синус от x
Chi(x)
Интегральный гиперболический косинус от x
В выражениях можно применять следующие операции:
Действительные числа
вводить в виде 7. 3
— возведение в степень
x + 7
— сложение
x — 6
— вычитание
15/7
— дробь
Другие функции:
asec(x)
Функция — арксеканс от x
acsc(x)
Функция — арккосеканс от x
sec(x)
Функция — секанс от x
csc(x)
Функция — косеканс от x
floor(x)
Функция — округление x в меньшую сторону (пример floor(4.5)==4.0)
ceiling(x)
Функция — округление x в большую сторону (пример ceiling(4.5)==5.0)
sign(x)
Функция — Знак x
erf(x)
Функция ошибок (или интеграл вероятности)
laplace(x)
Функция Лапласа
asech(x)
Функция — гиперболический арксеканс от x
csch(x)
Функция — гиперболический косеканс от x
sech(x)
Функция — гиперболический секанс от x
acsch(x)
Функция — гиперболический арккосеканс от x
Постоянные:
pi
Число «Пи», которое примерно равно ~3. 14159..
e
Число e — основание натурального логарифма, примерно равно ~2,7183..
i
Комплексная единица
oo
Символ бесконечности — знак для бесконечности
Сколько будет 7 в 18-й степени?
Итак, вы хотите знать, сколько будет 7 в 18-й степени, не так ли? В этой статье мы объясним, как именно выполнить математическую операцию под названием «возведение 7 в степень 18». Это может показаться фантастическим, но мы объясним это без жаргона! Давай сделаем это.
Что такое возведение в степень?
Давайте сначала зафиксируем наши термины, а затем посмотрим, как вычислить, сколько будет 7 в 18-й степени.
Когда мы говорим об возведении в степень, все, что мы на самом деле имеем в виду, это то, что мы умножаем число, которое мы называем 9) для обозначения показателя степени. Знак вставки полезен в ситуациях, когда вы не хотите или не нуждаетесь в использовании надстрочного индекса.
Итак, мы упомянули, что возведение в степень означает умножение базового числа само на себя для получения показателя степени число раз. Давайте посмотрим на это более наглядно:
7 в 18-й степени = 7 x … x 7 (18 раз)
Итак, каков ответ?
Теперь, когда мы объяснили теорию, лежащую в основе этого, давайте поработаем над числами и выясним, чему равно 7 в 18-й степени:
7 в степени 18 = 7 18 = 1 628 413 597 910 449
Почему мы вообще используем возведение в степень, такое как 7 18 ? Что ж, нам намного проще писать умножения и выполнять математические операции как с большими, так и с маленькими числами, когда вы работаете с числами с большим количеством конечных нулей или большим количеством десятичных знаков.
Надеюсь, эта статья помогла вам понять, как и почему мы используем возведение в степень, и дала вам ответ, который вы изначально искали. Теперь, когда вы знаете, что такое 7 в 18-й степени, вы можете продолжить свой веселый путь.
Не стесняйтесь поделиться этой статьей с другом, если вы считаете, что она поможет ему, или перейдите вниз, чтобы найти еще несколько примеров.
Процитируйте, дайте ссылку или ссылку на эту страницу
Если вы нашли этот контент полезным в своем исследовании, пожалуйста, сделайте нам большую услугу и используйте приведенный ниже инструмент, чтобы убедиться, что вы правильно ссылаетесь на нас, где бы вы его ни использовали. Мы очень ценим вашу поддержку!
«Сколько будет 7 в 18-й степени?». VisualFractions.com . По состоянию на 1 апреля 2023 г. http://visualfractions.com/calculator/exponent/what-is-7-to-the-18th-power/.
«Сколько будет 7 в 18-й степени?». VisualFractions.com , http://visualfractions.com/calculator/exponent/what-is-7-to-the-18th-power/. По состоянию на 1 апреля 2023 г.
Сколько будет 7 в 18-й степени?. VisualFractions.com. Получено с http://visualfractions.com/calculator/exponent/what-is-7-to-the-18th-power/.
Калькулятор возведения в степень
Хотите найти решение еще одной задачи? Введите число и мощность ниже и нажмите «Рассчитать».
Вычисление возведения в степень
Случайный список примеров возведения в степень
Если вы добрались до этого места, вам должно быть ДЕЙСТВИТЕЛЬНО нравится возведение в степень! Вот несколько случайных вычислений:
Сколько будет 79 в 12-й степени?
Сколько будет 66 в 19-й степени?
Сколько будет 40 в 33-й степени?
Сколько будет 32 в 34-й степени?
Сколько будет 76 в 39-й степени?
Сколько будет 38 в 81-й степени?
Сколько будет 9 в 40-й степени?
Сколько будет 17 в 70-й степени?
Сколько будет 82 в 69-й степени?
Сколько будет 24 в 50-й степени?
Сколько будет 89 в 90-й степени?
Сколько будет 42 в 10-й степени?
Сколько будет 65 в 37-й степени?
Сколько будет 3 в 16-й степени?
Сколько будет 15 в 91-я сила?
Сколько будет 67 в 72-й степени?
Сколько будет 37 в 19-й степени?
Сколько будет 61 в 35-й степени?
Сколько будет 2 в 84-й степени?
Сколько будет 98 в 54-й степени?
Сколько будет 79 в 37-й степени?
Сколько будет 68 в 8-й степени?
Сколько будет 54 в 36-й степени?
Сколько будет 56 в 61-й степени?
Сколько будет 72 в 31-й степени?
Сколько будет 29 в 60-й степени?
Сколько будет 86 в 22-й степени?
Сколько будет 31 в 25-й степени?
Сколько будет 54 в 85-й степени?
Сколько будет 28 в 72-й степени?
Сколько будет 14 в 13-й степени?
Сколько будет 38 в 68-й степени?
Сколько будет 28 в 89-й степени?
Сколько будет 88 в 76-й степени?
Сколько будет 80 в 63-й степени?
Сколько будет 54 в 44-й степени?
Сколько будет 71 в 60-й степени?
Сколько будет 33 в 8-й степени?
Сколько будет 40 в 21-й степени?
Сколько будет 26 в 45-й степени?
Сколько будет 13 в 28-й степени?
Сколько будет 32 в 54-й степени?
Сколько будет 22 в 74-й степени?
Сколько будет 75 в 22-й степени?
Сколько будет 88 в 93-й степени?
Сколько будет 17 в 89-й степени?
Сколько будет 90 в 23-й степени?
Сколько будет 29 в 32-й степени?
Сколько будет 62 в 73-й степени?
Сколько будет 97 в 7-й степени?
Сколько будет 12 в 58-й степени?
Сколько будет 44 в 15-й степени?
Сколько будет 72 в 93-й степени?
Сколько будет 11 в 65-й степени?
Сколько будет 22 в 19-й степени?
Сколько будет 85 в 31-й степени?
Сколько будет 85 в 73-й степени?
Сколько будет 11 в 16-й степени?
Сколько будет 18 в 83-й степени?
Сколько будет 55 в 84-й степени?
Сколько будет 58 в 44-й степени?
Сколько будет 64 в 30-й степени?
Сколько будет 24 в 36-й степени?
Сколько будет 59 в 51 степени?
Сколько будет 22 в 28-й степени?
Сколько будет 25 в 99-й степени?
Сколько будет 52 в 5-й степени?
Сколько будет 39 в 7-й степени?
Сколько будет 65 в 60-й степени?
Сколько будет 60 в 73-й степени?
Сколько будет 14 в 58-й степени?
Сколько будет 25 в 77-й степени?
Сколько будет 48 в 26-й степени?
Сколько будет 11 в 15-й степени?
Сколько будет 69 в сотой степени?
Сколько будет 77 в 92-й степени?
Сколько будет 21 в 87-й степени?
Сколько будет 33 в 15-й степени?
Сколько будет 54 в 93-й степени?
Сколько будет 71 в 93-й степени?
Сколько будет 46 в 18-й степени?
Сколько будет 3 в 49-й степени?
Сколько будет 36 в 78-й степени?
Сколько будет 29 в 31-й степени?
Сколько будет 88 в 79-й степени?
Сколько будет 87 в 63-й степени?
Сколько будет 40 на 89сила?
Сколько будет 79 в 63-й степени?
Сколько будет 59 в 51-й степени?
Сколько будет 82 в 60-й степени?
Сколько будет 32 в 25-й степени?
Сколько будет 33 в 91-й степени?
Сколько будет 19 в 73-й степени?
Сколько будет 53 в 15-й степени?
Сколько будет 95 в 63-й степени?
Сколько будет 95 в 64-й степени?
Сколько будет 8 в 25-й степени?
Сколько будет 20 в 42-й степени?
Сколько будет 76 в 34-й степени?
Сколько будет 91 в 99-й степени?
Что такое 833 в 3-й степени?
18 силовой стол
Вы ищете больше числовых диаграмм, используйте этот калькулятор
Power Table Generator
Калькулятор мощности
Преобразование экспоненты в число Установите флажок, чтобы преобразовать экспоненциальный результат в число. Число Поднятый силой Равно
Наша компания предлагает различные варианты оплаты приобретаемых вами товаров и услуг. Стоимость услуг типографии на сайте указана с учетом НДС 20%. Цены в магазине полиграфии, а также рассчитанные в профессиональном калькуляторе на сайте, в случае правильно проведенного расчета, являются актуальными и действительны в течение пяти рабочих дней. Получить дополнительную скидку вы можете, связавшись с нашими менеджерами.
Заказы на печать запускаются в производство только при наличии 100% предоплаты либо предоплаты в предварительно согласованном размере.
Исполнение заказов на условиях постоплаты вы можете обсудить с вашим менеджером.
Внимание! Для каждого заказа возможен только один способ оплаты на ваш выбор. Оплата заказа по частям различными способами невозможна.
Способы оплаты предоставляемые сервисом ЮKassa
Банковские карты:
Мир
Visa
MasterCard
Maestro
С карты «Мир» можно заплатить максимум 5000 a один раз и 15 000 a в месяц. С карты Visa, MasterCard или Maestro — максимум 250 000 a за один раз, в месяц — 500 000 a.
Электронные деньги:
Яндекс Деньги
За один раз можно заплатить из идентифицированного кошелька — до 250 000 a, из анонимного — до 15 000 a.
WebMoney
За один раз можно заплатить до 60 000 a. В месяц — до 150 000 a .
QIWI Wallet
За один раз можно заплатить до 250 000 a.
Интернет банкинг:
Сбербанк онлайн
Максимум в сутки: 10 000 a через мобильный банк и 150 000 a — через интернет-банк.
Альфа-Клик
За один раз можно заплатить до 60 000 a.
Интернет-банк Промсвязьбанка
За один раз можно заплатить до 60 000 a.
MasterPass
За один раз можно заплатить до 250 000 a. В месяц — до 500 000 a .
Баланс телефона:
Билайн
За один раз можно заплатить до 14 000 a, в сутки — до 15 000 a.
Мегафон
За один раз можно заплатить до 14 000 a, в сутки — до 15 000 a.
МТС
За один раз можно заплатить до 14 999 a, в сутки — до 30 000 a.
Безналичный расчет
При онлайн заказе в магазине полиграфии счет на оплату будет выписан автоматически по окончании оформления заказа. При оформлении заказа у менеджера счет будет выставлен после согласования условий поставки. Внимание! Счет действителен в течение пяти рабочих дней.
Любую интересующую вас информацию о способах оплаты вы можете уточнить у наших менеджеров по телефонам: + 7 (812) 702-19-60.
Наши реквизиты:
ООО «СИНЭЛ» ИНН: 7802093323 КПП: 780201001 Юридический адрес: 194223, г. Санкт-Петербург, ул. Курчатова, д.10 Почтовый адрес: 194223, г. Санкт-Петербург, ул. Курчатова, д. 10 Р/с: 40702810813000001280 в Ф. ОПЕРУ БАНКА ВТБ (ПАО) в Санкт-Петербурге, г. Санкт-Петербург К/с: 30101810200000000704 БИК: 044030704 ОКПО: 27410482 ОКАТО: 40265562000 ОКВЭД: 22. 2 ОКФС: 16 ОКОПФ: 65 ОГРН: 1037804022960
Дешевая печать визиток в СПб | Онлайн-калькулятор цен
Как заказать печать визиток?
Вы уже заказывали печать комплекта визиток и хотите сделать похожий тираж?
Сразу обращайтесь к нашим менеджерам. Они рассчитают стоимость производства по Вашему образцу.
Не знаете, какой выбрать вариант визитки?
Читайте дальше. Это поможет Вам определиться с материалом, способом печати и постпечатной обработки.
Не хотите долго читать? 🙂 Свяжитесь с нами в два клика
Калькулятор стоимости
Какие способы изготовления визиток вы можете предложить?
Цифровой способ печати
Обеспечивает низкую себестоимость при малых и средних тиражах, максимально быстрые сроки изготовления, возможность печати на широком спектре дизайнерских бумаг и картонов, а также синтетических материалах и пластиках.
Заказать цифровой тираж – хорошая идея, если…
вам нужна небольшая партия визиток в сжатые сроки.
Офсетный способ
Идеальный вариант для изготовления партий визиток, предназначенных для массовой раздачи или распространения по почтовым ящикам.
Заказать офсетный тираж визиток – хорошая идея, если вам необходимы визитки в больших количествах:
для участия в выставочных мероприятиях, конференциях, бизнес-встречах и т.д.,
для размещения на информационных стойках и стойках ресепшн,
для активной работы отдела продаж.
Шелкография
Позволяет изготавливать представительские визитные карточки из материалов, которые недоступны другим способам печати.
Непрозрачность красок и возможность печати толстым слоем позволяет получить яркие, контрастные цвета на темных материалах, а также приятный на ощупь рельеф изображения.
необходимо подчеркнуть индивидуальность владельца визитки, его статус или солидность организации,
Вы хотели бы отразить в визитке специфику своего производства.
Мы напечатали
Я хочу сделать мою визитку запоминающейся!
При разработке дизайна важно учесть детали: правильно подобрать шрифт и цвет, гармонично соотнести текст с графикой, продумать расположение элементов.
Нередко именно по визитке первый раз составляют впечатление о держателе, поэтому поручите разработку макета профессионалам.
Мы можем выполнить не только печать визиток, но и в кратчайший срок разработать их макет в нашей дизайн-студии.
Добавьте тактильных ощущений благодаря приятной на ощупь бархатистой ламинации. Придайте нестандартную форму, заказав фигурную вырубку.
Тиснение фольгой
Конгрев
Выборочный лак
Лак и выборочный УФ лак может быть с блестками, текстурой, глянцевый и матовый.
Печать Pantone
Объемное 3D фольгирование
Фигурная вырубка
Могу я заказать сигнальный образец?
Конечно! Если он вас полностью устроит, и тираж будет печататься с того же макета, то мы учтем часть его цены в стоимости тиража.
Если нет возможности посетить наш офис лично, мы доставим образец курьерской службой. Печать тиража визиток осуществляется только после согласования с заказчиком, поэтому Вы можете быть уверенны, что готовая продукция Вас точно устроит.
Почему мне стоит разместить заказ на печать визиток в вашей типографии?
Более 25 лет мы изготавливаем различные продукты для бизнеса и рекламы, в том числе, Вы сможете заказать визитные карточки в СПб или с доставкой в любой регион России.
У нас работает квалифицированный персонал, есть все необходимое собственное оборудование. Поток (объем) заказов позволяет нам получать низкие цены у поставщиков расходных материалов и бумаги, сохраняя лучшее соотношение цены и качества даже с учетом доставки в регионы.
Готовые решения для быстрого заказа
Заказать визитки недорого можно в компании «Синэл», которая предлагает клиенту вариации из немелованной бумаги, мелованного, дизайнерского, тонированного картона.
Размер для своего изделия можно выбрать из двух стандартов — 90×50 мм или 85×55 мм. А выполнение печати возможно как в одностороннем, так и в двухстороннем формате.
Качественные, красочные и оригинальные изделия от наших мастеров обязательно привнесут свой вклад в развитие вашего бизнеса. Приобрести товар данной категории можно в тираже от 1 шт. Количество изделий в тираже не ограничивается.
Заказать визитки недорого можно на сайте в несколько кликов. Просто, быстро и выгодно — это наш бонус для любимых клиентов.
Подбор параметров
Визитки евростандарт 85×55мм односторонние
Артикул: 9674
85×55мм. Двухсторонний мелованный визиточный картон 300 г/кв.м, цветная (4+0) печать с одной стороны
Плотность бумаги, г/м.кв: 300
Визитки стандарт 90×50мм односторонние
Артикул: 9671
90×50мм. Двухсторонний мелованный визиточный картон 300 г/кв.м, цветная (4+0) печать с одной стороны
Плотность бумаги, г/м.кв: 300
Визитки евростандарт 85×55мм двухсторонние
Артикул: 9673
85×55мм. Двухсторонний мелованный визиточный картон 300 г/кв.м, цветная (4+4) печать с двух сторон
Плотность бумаги, г/м.кв: 300
Визитки стандарт 90×50мм двухсторонние
Артикул: 9672
90×50мм. Двухсторонний мелованный визиточный картон 300 г/кв.м, цветная (4+4) печать с двух сторон
Плотность бумаги, г/м. кв: 300
Телефон
*
Примечание
*
Нажимая кнопку «Отправить», я даю свое согласие на обработку моих персональных данных, в соответствии с Федеральным законом от 27.07.2006 года №152-ФЗ «О персональных данных», на условиях и для целей, определенных в Согласии на обработку персональных данных
*
Методом электронно-ионных уравненийсоставьте полные уравнения реакций, учитывая, что либо окислитель, либо восстановитель являются также и средой. Обоснуйте на основании стандартных окислительно-восстановительных потенциалов возможность протекания данных реакций
Ангелина Эдуардовна
Предметы: Литература, Литературное чтение, Русский язык
Так как в реакции участвует кислота, то среда кислая, соответственно, в полуреакциях можно использовать только ионы водорода.
Для обоснования возможности протекания данных реакций необходимо рассчитать изменение стандартного потенциала по формуле:
В таблице стандартных окислительно-восстановительных потенциалов полуреакций находим соответствующие потенциалы:
Следовательно, реакция возможна при стандартных условиях.
Здесь мы аналогично используем ионы водорода в полуреакциях, так как среда кислая. 2 − 6x
Ответ данНаталья Сергеевна
Предметы: Математика, Геометрия, Алгебра, Окружающий мир, Начальные классы, Литературное чтение, Подготовка к ОГЭ, Русский язык
Алгебра7 класс
У
Ученик
Здравствуйте! Помогите, пожалуйста, с задачей, не понимаю как её решать. Три белки делили орехи. Сначала первая раздала половину своих орехов двум другим поровну. Потом вторая, а затем и третья белка поступили так же. В итоге у них стало 19, 14 и 8 орехов соответственно. Сколько было у каждой белки …
Ответ данНаталья Васильевна
Предметы: Математика, Подготовка к школе, Окружающий мир, Начальные классы, Литературное чтение, Русский язык, Онлайн няня
Литература7 класс
У
Ученик
помогите пожалуйста, очень надо!!Самый запоминающийся эпизод в сказе «богиня в шинели» и о чём заставляет задуматься данное произведение? (И.М. Ермаков) буду очень благодарна!!
Ответ данНаталья Васильевна
Предметы: Математика, Подготовка к школе, Окружающий мир, Начальные классы, Литературное чтение, Русский язык, Онлайн няня
Химия8 класс
У
Ученик
Здравствуйте! Помогите, пожалуйста, с решением задачи. 2 = 5,5𝑥 + 3 сначала графически, азатем с помощью формулы корней.
Ответ данНаталья Васильевна
Предметы: Математика, Подготовка к школе, Окружающий мир, Начальные классы, Литературное чтение, Русский язык, Онлайн няня
Всеобщая история7 класс
М
Макар
Помогите пожалуйста. сравнение уровня социально экономического развития германских государств с развитием англии или франции и укажи на основе различия.
Ответ данНаталья Васильевна
Предметы: Математика, Подготовка к школе, Окружающий мир, Начальные классы, Литературное чтение, Русский язык, Онлайн няня
Геометрия8 класс
М
Матвей
Из вершины 𝐵 треугольника 𝐴𝐵𝐶 проведеныбиссектриса 𝐵𝐸 и высота 𝐵𝐻, равная 12, причем высота делитсторону 𝐴𝐶 на отрезки 𝐴𝐻 и 𝐻𝐶, равные 9 и 16 соответственно.Найдите стороны треугольника и отрезки, на которые биссектриса делит сторону 𝐴𝐶.
Ответ данНаталья Сергеевна
Предметы: Математика, Геометрия, Алгебра, Окружающий мир, Начальные классы, Литературное чтение, Подготовка к ОГЭ, Русский язык
Геометрия8 класс
М
Матвей
Периметр треугольника 𝐴𝐵𝐶 равен 60 см. На сторонах𝐴𝐵, 𝐵𝐶, 𝐴𝐶 лежат точки 𝐷, 𝐸, 𝐹 соответственно так, что 𝐷𝐵𝐸𝐹 −ромб. Найдите стороны 𝐴𝐵 и 𝐵𝐶 треугольника, если 𝐴𝐹 = 10,𝐹𝐶 = 15.
Ответ дан
Тепаева Мария
История России6 класс
И
Ирина
Не могу найти значимые события в 982 году ? по истории 6 класс
Ответ данИнна Игоревна
Предметы: Математика, Подготовка к школе, История России, Окружающий мир, Подготовка к ЕГЭ, Обществознание, Всеобщая история, ИЗО, МХК, Литературное чтение, Русский язык
Русский язык8 класс
М
Матвей
Найдите предложения с несогласованным определением, запишите только номера предложений: 1. Солнце гуляло по мокрым от вчерашнего дождя жнивьям. 2. Я не мог забыть девушку с синими глазами. 3. Дальше по оврагу еще лежал снег в желтых, в синих пятнах. 4. После душной ночи потянул на рассвете свежий ве…
Ответ данНаталья Васильевна
Предметы: Математика, Подготовка к школе, Окружающий мир, Начальные классы, Литературное чтение, Русский язык, Онлайн няня
Как уравнивать в химии.
Как расставлять коэффициенты в химических уравнениях
Часть I
1. Закон Ломоносова-Лавуазье – закон сохранения массы веществ:
2. Уравнения химической реакции – это условная запись химической реакции с помощью химических формул и математических знаков.
3. Химическое уравнение должно соответствовать закону сохранения массы веществ, что достигается расстановкой коэффициентов в уравнении реакции.
4. Что показывает химическое уравнение? 1) Какие вещества вступают в реакцию. 2) Какие вещества образуются в результате. 3) Количественные отношения веществ в реакции, т. е. количества реагирующих и образующихся веществ в реакции. 4) Тип химической реакции.
5. Правила расстановки коэффициентов в схеме химической реакции на примере взаимодействия гидроксида бария и фосфорной кислоты с образованием фосфата бария и воды. а) Запишите схему реакции, т. е. формулы реагирующих и образующихся веществ:
б) начинайте уравнивать схему реакции с формулы соли (если она имеется). При этом помните, что несколько сложных ионов в составе основания или соли обозначаются скобками, а их число – индексами за скобками:
в) водород уравняйте в предпоследнюю очередь:
г) кислород уравняйте последним – это индикатор верной расстановки коэффициентов. Перед формулой простого вещества возможна запись дробного коэффициента, после чего уравнение необходимо переписать с удвоенными коэффициентами.
Часть II
1. Составьте уравнения реакций, схемы которых:
2. Напишите уравнения химических реакций:
3. Установите соответствие между схемой и суммой коэффициентов в химической реакции.
4. Установите соответствие между исходными веществами и продуктами реакции.
5. Что показывает уравнение следующей химической реакции:
1) Вступили в реакцию гидроксид меди и соляная кислота; 2) Образовались в результате реакции соль и вода; 3) Коэффициенты перед исходными веществами 1 и 2.
6. С помощью следующей схемы составьте уравнение химической реакции, используя удвоение дробного коэффициента:
7. Уравнение химической реакции: 4P+5O2=2P2O5 показывает количество вещества исходных веществ и продуктов, их массу или объём: 1) фосфора – 4 моль или 124 г; 2) оксида фосфора (V) – 2 моль, 284 г; 3) кислорода – 5 моль или 160 л.
Для того чтобы выяснить, как уравнять химическое уравнение, для начала следует узнать предназначение данной науки.
Определение
Химия изучает вещества, их свойства, а также превращения. В случае если не наблюдается изменения окраски, выпадения осадка, выделения газообразного вещества, то не происходит никакого химического взаимодействия.
Например, при обработке напильником железного гвоздя металл просто превращается в порошок. В этом случае никакой химической реакции не происходит.
Прокаливание перманганата калия сопровождается образованием оксида марганца (4), выделением кислорода, то есть наблюдается взаимодействие. При этом возникает вполне закономерный вопрос о том, как правильно уравнивать химические уравнения. Разберем все нюансы, связанные с подобной процедурой.
Специфика химических превращений
Любые явления, которые сопровождаются изменением качественного и количественного состава веществ, относятся к химическим превращениям. В молекулярном виде процесс сгорания железа в атмосфере можно выразить с помощью знаков и символов.
Методика расстановки коэффициентов
Как уравнивать коэффициенты в химических уравнениях? В курсе химии средней школы разбирается метод электронного баланса. Рассмотрим процесс более подробно. Для начала в исходной реакции необходимо расставить степени окисления у каждого химического элемента.
Существуют определенные правила, по которым их можно определить у каждого элемента. В простых веществах степени окисления будут равны нулю. В бинарных соединениях у первого элемента она положительна, соответствует высшей валентности. У последнего данный параметр определяется путем вычитания номера группы из восьми и имеет знак «минус». В формулах, состоящих их трех элементов, есть свои нюансы вычисления степеней окисления.
Для первого и последнего элемента порядок аналогичен определению в бинарных соединениях, а для вычисления центрального элемента составляется уравнение. Сумма всех показателей должна быть равна нулю, исходя из этого, вычисляется показатель для среднего элемента формулы.
Продолжим разговор о том, как уравнивать химические уравнения методом электронного баланса. После того как степени окисления будут поставлены, можно определять те ионы либо вещества, которые в ходе химического взаимодействия изменили их значение.
Знаками «плюс» и «минус» необходимо указать количество электронов, которые были приняты (отданы) в процессе химического взаимодействия. Между полученными цифрами находят наименьшее общее кратное.
При делении его на принятые и отданные электроны получают коэффициенты. Как уравнять химическое уравнение? Полученные в балансе цифры нужно поставить перед соответствующими формулами. Обязательным условием является проверка количества каждого элемента в левой и правой части. Если коэффициенты расставлены правильно, их число должно быть одинаковым.
Закон сохранения массы веществ
Рассуждая над тем, как уравнять химическое уравнение, необходимо использовать именно этот закон. Учитывая, что масса тех веществ, которые вступили в химическую реакцию, равна массе образующихся продуктов, становится возможным постановка коэффициентов перед формулами. Например, как уравнять химическое уравнение, если вступают во взаимодействие простые вещества кальций и кислород, а после завершения процесса получается оксид?
Чтобы справиться с поставленной задачей, необходимо учитывать, что кислород является двухатомной молекулой с ковалентной неполярной связью, поэтому его формула записывается в следующем виде — О2. В правой части при составлении оксида кальция (СаО) учитывают валентности каждого элемента.
Сначала необходимо проверить количество кислорода в каждой части уравнения, так как оно отличается. По закону сохранения массы веществ перед формулой продукта нужно поставить коэффициент 2. Далее проводится проверка кальция. Для того чтобы он был уравнен, перед исходным веществом ставим коэффициент 2. В итоге получаем запись:
2Са+О2=2СаО.
Разбор реакции методом электронного баланса
Как уравнивать химические уравнения? Примеры ОВР помогут ответить на данный вопрос. Допустим, что необходимо методом электронного баланса расставить коэффициенты в предложенной схеме:
CuO + Н2=Cu + Н2О.
Для начала у каждого из элементов в исходных веществах и продуктах взаимодействия расставим значения степеней окисления. Получим следующий вид уравнения:
Cu(+2)О(-2)+Н2(0)=Cu(0)+Н2(+)О(-2).
Показатели изменились у меди и водорода. Именно на их основе будем составлять электронный баланс:
Cu(+2)+2е=Cu(0) 1 восстановитель, окисление;
Н2(0)-2е=2Н(+) 1 окислитель, восстановление.
Исходя из коэффициентов, полученных в электронном балансе, получаем следующую запись предложенного химического уравнения:
CuO+Н2=Cu+Н2О.
Возьмем еще один пример, который предполагает постановку коэффициентов:
Н2+О2=Н2О.
Для того чтобы уравнять на основе закона сохранения веществ данную схему, необходимо начать с кислорода. Учитывая, что вступала в реакцию двухатомная молекула, перед формулой продукта взаимодействия необходимо поставить коэффициент 2.
2Н2+О2=2Н2О.
Заключение
На основании электронного баланса можно расставлять коэффициенты в любых химических уравнениях. Выпускникам девятых и одиннадцатых классов образовательных учреждений, выбирающим экзамен по химии, в одном из заданий итоговых тестов предлагают подобные задания.
Решение уравнений химический реакций вызывают затруднения у немалого количества учеников средней школы во-многом благодаря большому разнообразию участвующих в них элементов и неоднозначности их взаимодействия. Но так как основная часть курса общей химии в школе рассматривает именно взаимодействие веществ на основе их уравнений реакций, то ученикам необходимо обязательно ликвидировать пробелы в данной области и научиться решать химические уравнения, чтобы избежать проблем с предметом в дальнейшем.
Уравнением химической реакции называется символьная запись, отображающая взаимодействующие химические элементы, их количественное соотношение и получающиеся в результате взаимодействия вещества. Данные уравнения отражают сущность взаимодействия веществ с точки зрения атомно-молекулярного или электронного взаимодействия.
В самом начале школьного курса химии учат решать уравнения на основе понятия валентности элементов периодической таблицы. На основе данного упрощения рассмотрим решение химического уравнения на примере окисления алюминия кислородом. Алюминий, взаимодействуя с кислородом, образует оксид алюминия. Обладая указанными исходными данными составим схему уравнения.
Al + O 2 → AlO
В данном случае мы записали примерную схему химической реакции, которая лишь частично отражает ее сущность. В левой части схемы записываются вещества, вступающую в реакцию, а в правой результат их взаимодействия. Кроме того, кислород и другие типичные окислители, обычно записываются правее металлов и других восстановителей в обоих частях уравнения. Стрелка показывает направление реакции.
Чтобы данная составленная схема реакции приобрела законченный вид и соответствовала закону сохранения массы веществ, необходимо:
Проставить индексы в правой части уравнения у вещества, получившегося в результате взаимодействия.
Уровнять количество участвующих в реакции элементов с количеством получившегося вещества в соответствии с законом сохранения массы веществ.
Начнем с приостановки индексов в химической формуле готового вещества. Индексы устанавливаются в соответствии с валентностью химических элементов. Валентностью называют способность атомов образовывать соединения с другими атомами за счет соединения их неспаренных электронов, когда одни атомы отдают свои электроны, а другие присоединяют их себе на внешний энергетический уровень. Принято считать, что валентность химического элемента определяет его группой (колонкой) в периодической таблице Менделеева. Однако на практике взаимодействие химических элементов происходит гораздо сложнее и разнообразнее. Например, атом кислорода во всех реакциях имеет валентность Ⅱ, несмотря на то, что в периодической таблице находится в шестой группе.
Чтобы помочь вам сориентироваться в этом многообразии, предлагаем вам следующий небольшой справочный помощник, который поможет определить валентность химического элемента. Выберите интересующий вас элемент и вы увидите возможные значения его валентности. В скобках указаны редкие для выбранного элемента валентности.
Вернемся к нашему примеру. Запишем в правой части схемы реакции сверху над каждым элементом его валентность.
Для алюминия Al валентность будет равна Ⅲ, а для молекулы кислорода O 2 валентность равна Ⅱ. Находим наименьшее общее кратное к этим числам. Оно будет равно шести. Делим наименьшее общее кратное на валентность каждого элемента и получаем индексы. Для алюминия шесть делим на валентность получаем индекс 2, для кислорода 6/2=3. Химическая формула оксида алюминия, полученного в результате реакции, примет вид Al 2 O 3 .
Al + O 2 → Al 2 O 3
После получения правильной формулы готового вещества необходимо проверить и в большинстве случаев уравнять правые и левые части схемы согласно закона сохранения массы, так как продукты реакции образуются из тех же атомов, которые изначально входили в состав исходных веществ, участвующих в реакции.
Закон сохранения массы гласит, что количество атомов вступивших в реакцию должно равняться количеству атомов получившихся в результате взаимодействия. В нашей схеме во взаимодействии участвуют один атом алюминия и два атома кислорода. В результате реакции получаем два атома алюминия и три кислорода. Очевидно, что схему необходимо уровнять, используя коэффициенты для элементов и вещества, чтобы соблюдался закон сохранения массы.
Уравнивание выполняют также через нахождение наименьшего общего кратного, которое находится между элементами, обладающими наибольшими индексами. В нашем примере это будет кислород с индексом в правой части равным 3 и в левой части равным 2. Наименьшее общее кратное и в этом случае будет равно 6. Теперь разделим наименьшее общее кратное на значение наибольшего индекса в левой и правой частях уравнения и получим следующие индексы для кислорода.
Al + 3∙O 2 → 2∙Al 2 O 3
Теперь остается уравнять только алюминий в правой части. Для этого в левую часть поставим коэффициент 4.
4∙Al + 3∙O 2 = 2∙Al 2 O 3
После расстановки коэффициентов уравнение химической реакции соответствует закону сохранения массы и между его левой и правой частями можно поставить знак равенства. Расставленные коэффициенты в уравнении обозначают число молекул веществ, участвующих в реакции и получающихся в результате нее, или соотношение данных веществ в молях.
После выработки навыков решения химических уравнений на основе валентностей взаимодействующих элементов, школьный курс химии знакомит с понятием степени окисления и теорией окислительно-восстановительных реакций. Данный тип реакций является наиболее распространенным и в дальнейшем химические уравнения чаще всего решают на основе степеней окисления взаимодействующих веществ. О том, рассказано в соответствующей статье на нашем сайте.
Для характеристики определенной химической реакции необходимо уметь составить запись, которая будет отображать условия протекания химической реакции, показывать какие вещества вступили в реакцию, а какие образовались. Для этого используют схемы химических реакций.
Схема химической реакции – условная запись, показывающая, какие вещества вступают в реакцию, какие продукты реакции образуются, а также условия протекания реакции
Рассмотрим в качестве примера реакцию взаимодействия угля и кислорода. Схема данной реакции записывается следующим образом:
С + О2 → СО2.
уголь взаимодействует с кислородом с образованием углекислого газа
Углерод и кислород – в данной реакции реагенты, а полученный углекислый газ – продукт реакции. Знак «→ » обозначает протекание реакции. Часто над стрелкой пишут условия, при которых происходит реакция
Например, знак « t° → » обозначает, что реакция протекает при нагревании. Знак « Р → » обозначает давление, а знак « hv → » – что реакция протекает под действием света. Также над стрелкой могут указывать дополнительные вещества, участвующие в реакции. Например, « О2 → ».
Если в результате химической реакции образуется газообразное вещество, то в схеме реакции, после формулы этого вещества записывают знак «→ ». Если при протекании реакции образуется осадок, его обозначают знаком «→ ».
Например, при нагревании порошка мела (он содержит вещество с химической формулой CaCO3), образуются два вещества: негашеная известь CaO и углекислый газ.
СaCO3 t° → CaO + CO2 .
В тех случаях, когда и реагенты и продукты реакции, например, являются газами, знак «» не ставят. Так, природный газ, в основном состоит из метана Ch5, при его нагревании до 1500°С он превращается в два других газа: водород Н2 и ацетилен С2Н2. Схема реакции записывается так:
Ch5 t° → C2h3 + h3.
Важно не только уметь составлять схемы химических реакций, но и понимать, что они обозначают. Рассмотрим, еще одну схему реакции:
h3O эл.ток → Н2 + О2
Данная схема означает, что под действием электрического тока, вода разлагается на два простых газообразных вещества: водород и кислород. Схема химической реакции является подтверждением закона сохранения массы и показывает, что химические элементы во время химической реакции не исчезают, а только перегруппировываются в новые химические соединения.
Уравнения химических реакций
Согласно закону сохранения массы исходная масса продуктов всегда равна массе полученных реагентов. Количество атомов элементов до и после реакции всегда одинаковое, атомы только перегруппировываются и образуют новые вещества.
Вернемся к схемам реакций, записанным ранее:
СaCO3 t° → CaO + CO2; С + О2 СО2.
В данных схемах реакций знак «→ » можно заменить на знак «=», так как видно, что количество атомов до и после реакций одинаковое. Записи будут иметь следующий вид:
СaCO3 = CaO + CO2; С + О2 = СО2.
Именно такие записи называют уравнениями химических реакций, то есть, это – записи схем реакций, в которых количество атомов до и после реакции одинаковое.
Уравнение химической реакции – условная запись химической реакции посредством химических формул, которая соответствует закону сохранения массы вещества
Если мы рассмотрим другие, приведенные ранее схемы уравнений, можно заметить, что на первый взгляд, закон сохранения массы в них не выполняется:
Ch5 t° → C2h3 + h3.
Видно, что в левой части схемы, атом углерода один, а в правой – их два. Атомов водорода поровну и в левой и правой частях их по четыре. Превратим данную схему в уравнение. Для этого необходимо уравнять количество атомов углерода. Уравнивают химические реакции при помощи коэффициентов, которые записывают перед формулами веществ.
Очевидно, чтобы количество атомов углерода стало одинаковым слева и справа, в левой части схемы, перед формулой метана, необходимо поставить коэффициент 2:
2Ch5 t° → C2h3 + h3
Видно, что атомов углерода слева и справа теперь поровну, по два. Но теперь неодинаково количество атомов водорода. В левой части уравнения их 2∙4 = 8. В правой части уравнения атомов водорода 4 (два из них в молекуле ацетилена, и еще два – в молекуле водорода). Если поставить коэффициент перед ацетиленом, нарушится равенство атомов углерода. Поставим перед молекулой водорода коэффициент 3:
2Ch5 = C2h3 + 3h3
Теперь количество атомов углерода и водорода в обеих частях уравнения одинаковое. Закон сохранения массы выполняется!
Рассмотрим другой пример. Схему реакции Na + h3O → NaOH + h3 необходимо превратить в уравнение.
В данной схеме различным является количество атомов водорода. В левой части два, а в правой – три атома. Поставим коэффициент 2 перед NaOH.
Na + h3O → 2NaOH + h3
Тогда атомов водорода в правой части станет четыре, следовательно, коэффициент 2 необходимо добавить и перед формулой воды:
Na + 2h3O → 2NaOH + h3
Уравняем и количество атомов натрия:
2Na + 2h3O = 2NaOH + h3
Теперь количество всех атомов до и после реакции одинаковое.
Таким образом, можно сделать вывод: чтобы превратить схему химической реакции в уравнение химической реакции, необходимо уравнять количество всех атомов, входящих в состав реагентов и продуктов реакции при помощи коэффициентов. Коэффициенты ставятся перед формулами веществ.
Подведем итоги об Уравнения химических реакций
Схема химической реакции – условная запись, показывающая, какие вещества вступают в реакцию, какие продукты реакции образуются, а также условия протекания реакции
В схемах реакций используют обозначения, указывающие на особенности их протекания
Уравнение химической реакции – условная запись химической реакции посредством химических формул, которая соответствует закону сохранения массы вещества
Схему химической реакции превращают в уравнение путем расстановки коэффициентов перед формулами веществ
Химическим уравнением можно назвать визуализацию химической реакции с помощью знаков математики и химических формул. Такое действие является отображением какой-либо реакции, в процессе которой появляются новые вещества.
Химические задания: виды
Химическое уравнение — это последовательность химических реакций. Они основываются на законе сохранения массы каких-либо веществ. Существует всего два вида реакций:
Соединения — к ним относятся (происходит замена атомов сложных элементов атомами простых реагентов), обмена (замещение составными частями двух сложных веществ), нейтрализации (реакция кислот с основаниями, образование соли и воды).
Разложения — образование двух и более сложных или простых веществ из одного сложного, но состав их более простой.
Химические реакции также можно разделить на типы: экзотермические (происходят с выделением теплоты) и эндотермические (поглощение теплоты).
Этот вопрос волнует многих учащихся. Мы предлегаем несколько простых советов, которые подскажут, как научиться решать химические уравнения:
Желание понять и освоить. Нельзя отступать от своей цели.
Теоретические знания. Без них невозможно составить даже элементарную формулу соединения.
Правильность записи химической задачи — даже малейшая ошибка в условии сведет к нулю все ваши усилия в ее решении.
Желательно, чтобы сам процесс решения химических уравнений был для вас увлекательным. Тогда химические уравнения (как решать их и какие моменты нужно запомнить, мы разберем в этой статье) перестанут быть для вас проблемными.
Задачи, которые решаются с использованием уравнений химических реакций
К таким задачам относятся:
Нахождение массы компонента по данной массе другого реагента.
Задания по комбинации «масса-моль».
Расчеты по комбинации «объем-моль».
Примеры с применением термина «избыток».
Расчеты с использованием реагентов, один из которых не лишен примесей.
Задачи на распад результата реакции и на производственные потери.
Задачи на поиск формулы.
Задачи, в которых реагенты предоставлены в виде растворов.
Задачи, содержащие смеси.
Каждый из этих видов задач включает в себя несколько подтипов, которые обычно подробно рассматриваются еще на первых школьных уроках химии.
Химические уравнения: как решать
Существует алгоритм, который помогает справиться с практически любым заданием из этой непростой науки. Чтобы понять, как правильно решать химические уравнения, нужно придерживаться определенной закономерности:
При записи уравнения реакции не забывать расставлять коэффициенты.
Определение способа, с помощью которого можно найти неизвестные данные.
Правильность применения в выбранной формуле пропорций или использование понятия «количество вещества».
Обратить внимание на единицы измерений.
В конце важно обязательно проверить задачу. В процессе решения вы могли допустить элементарную ошибку, которая повлияла на результат решения.
Основные правила составления химических уравнений
Если придерживаться правильной последовательности, то вопрос о том, что такое химические уравнения, как решать их, не будет вас волновать:
Формулы веществ, которые вступают в реакцию (реагенты), записываются в левой части уравнения.
Формулы веществ, которые образуются в результате реакции, записываются уже в правой части уравнения.
Составление уравнения реакции основывается на законе сохранения массы веществ. Следовательно, обе части уравнения должны быть равны, т. е. с одинаковым числом атомов. Достичь этого можно при условии правильной расстановки коэффициентов перед формулами веществ.
Расстановка коэффициентов в химическом уравнении
Алгоритм расстановки коэффициентов таков:
Подсчет в левой и правой части уравнения атомов каждого элемента.
Определение меняющегося количества атомов у элемента. Также нужно найти Н.О.К.
Получение коэффициентов достигается путем деления Н.О.К. на индексы. Обязательно проставить данные цифры перед формулами.
Следующим шагом является пересчет количества атомов. Иногда возникает необходимость в повторении действия.
Уравнивание частей химической реакции происходит с помощью коэффициентов. Расчет индексов производится через валентность.
Для успешного составления и решения химических уравнений необходимо учитывать физические свойства вещества, такие как объем, плотность, масса. Также нужно знать состояние реагирующей системы (концентрация, температура, давление), разбираться в единицах измерения данных величин.
Для понимания вопроса о том, что такое химические уравнения, как решать их, необходимо использование основных законов и понятий этой науки. Чтобы успешно вычислять подобные задачи, необходимо также вспомнить или освоить навыки математических операций, уметь совершать действия с числами. Надеемся, с нашими советами вам будет легче справляться с химическими уравнениями.
7 Крутые игры с балансировкой химических уравнений, в которые можно играть онлайн
Балансировка химических уравнений — это фундаментальное понятие в химии, которое помогает понять поведение различных химических веществ и то, как они реагируют друг с другом. Это важный шаг в прогнозировании исхода химической реакции и понимании стехиометрии реакции.
Однако понять и запомнить правила и методы балансировки химических уравнений может быть непросто. В этом случае онлайн-игры могут служить веселым и интерактивным способом помочь учащимся изучать и практиковать балансирование химических уравнений. Независимо от того, являетесь ли вы студентом, пытающимся сдать следующий тест, или любознательным учеником, стремящимся углубить свое понимание химии, веселые занятия и интерактивные онлайн-игры — идеальный способ сделать изучение химических реакций увлекательным и увлекательным.
Итак, приготовьтесь прокачать свои химические навыки с помощью этих эксклюзивных онлайн-игр, которые бросят вызов вашему разуму и помогут овладеть искусством балансировки химических уравнений. Берите лабораторный халат и защитные очки и окунитесь в мир химических реакций и онлайн-игр!
Облегчение понимания балансировки химических уравнений с помощью онлайн-игр
Благодаря множеству онлайн-игр на химию дети могут также принять участие в нескольких играх с балансировкой химических уравнений, которые помогут им лучше понять эту концепцию. Итак, вот несколько игр, в которые можно играть в классе или даже дома!
1. Игра «Химическая реакция» в Музее науки Чикаго:
Это образовательная игра, которая моделирует химические реакции и знакомит игроков с элементами, соединениями и химическими реакциями. В игре обычно участвуют игроки, смешивающие различные элементы и соединения для создания новых соединений и наблюдения за получающимися химическими реакциями. Игра может включать несколько уровней или настроек сложности, а также может предоставлять информацию и пояснения об элементах и их свойствах.
Воспитательное значение этой игры заключается в том, что она помогает игрокам понять фундаментальные принципы химии и то, как различные элементы и соединения взаимодействуют друг с другом. Он также предоставляет учащимся интерактивный и визуальный опыт обучения, который может помочь им лучше понять и запомнить материал.
Ссылка на игру
2. Outer Space Molecule Chase
Это игра, которая имитирует процесс создания молекул путем объединения атомов. В игре обычно участвуют игроки, которые перемещаются в пространстве, собирают различные атомы и используют их для создания молекул. Игра может включать несколько уровней или настроек сложности, а также может предоставлять информацию и пояснения об элементах и их свойствах.
Цель игры — помочь игрокам узнать о молекулах, их свойствах и способах их образования в увлекательной и увлекательной форме. Образовательное значение этой игры заключается в том, что она помогает учащимся узнать о концепции химических соединений и о том, как атомы объединяются в молекулы. Это также позволяет учащимся практиковаться в построении молекул, что может помочь им понять свойства и поведение различных соединений.
Ссылка на игру
3. Игра «Балансировка химических уравнений»
3. Игра «Балансировка химических уравнений» на TinyTap — это образовательная игра, предназначенная для того, чтобы помочь учащимся научиться составлять химические уравнения. Игра обычно представляет игроку несбалансированное химическое уравнение, и игрок должен использовать свои знания о химических реакциях и принципах сохранения массы, чтобы сбалансировать уравнение, регулируя коэффициенты реагентов и продуктов.
Затем игрок может проверить свою работу, представив уравнение, и игра сообщит, сбалансировано ли уравнение или нет. Образовательное использование этой игры должно помочь учащимся развить их понимание химических реакций и принципов сохранения массы.
Играя в игру, учащиеся могут практиковаться в балансировке уравнений и получать немедленную обратную связь о своей работе, которая может помочь им выявить и исправить любые ошибки, которые они могут совершать. Кроме того, игровой формат делает изучение химических уравнений более увлекательным и интерактивным, что может помочь повысить мотивацию учащихся и их вовлеченность в материал.
Ссылка на игру
4. PhET: балансировка химических уравнений
Игра балансировки химических уравнений на PhET представляет собой интерактивную симуляцию, предназначенную для того, чтобы помочь учащимся научиться составлять химические уравнения. Игра обычно предоставляет игроку интерактивный интерфейс, который позволяет ему манипулировать реагентами и продуктами в химическом уравнении, регулируя количество атомов или молекул каждого элемента.
После этого игрок может проверить свою работу, нажав кнопку, чтобы сбалансировать уравнение, и игра сообщит, сбалансировано уравнение или нет. Эта игра предназначена для того, чтобы помочь учащимся развить свое понимание химических реакций и принципов сохранения массы путем визуального представления атомов и молекул, участвующих в реакции.
Кроме того, интерактивный характер моделирования позволяет учащимся экспериментировать с различными способами уравновешивания уравнений, что может помочь им более эффективно идентифицировать и понимать основные концепции.
Ссылка на игру
5. Химия: игра с балансирующими уравнениями – Из грязи в князи-2020i
«Химия: игра с балансирующими уравнениями – Из грязи в князи-2020i» на Quia – это обучающая игра, предназначенная для помощи студенты узнают, как сбалансировать химические уравнения. Игра представляет собой формат множественного выбора, в котором учащимся предлагается химическое уравнение и четыре различных варианта его баланса.
Игрок должен использовать свои знания о химических реакциях и принципах сохранения массы, чтобы выбрать правильное сбалансированное уравнение из четырех вариантов. Игра также включает в себя систему начисления очков и функцию таблицы лидеров, которая может добавить элемент соревнования и мотивации для студентов.
Образовательное использование этой игры состоит в том, чтобы помочь учащимся лучше понять химические реакции и принципы сохранения массы, предоставляя учащимся возможность попрактиковаться в уравнениях баланса и получить немедленную обратную связь о своей работе.
Кроме того, формат множественного выбора позволяет учащимся проверить свое понимание уравнений баланса, что может помочь им выявить и исправить любые ошибки, которые они могут совершать. Формат игры с дополнительными функциями очков и таблицы лидеров делает изучение химических уравнений более увлекательным и интерактивным, что может помочь повысить мотивацию учащихся и их вовлеченность в материал.
Ссылка на игру
6. iwant2study: Балансировка химических уравнений
«Балансировка химических уравнений» предназначена для того, чтобы помочь пользователям научиться балансировать химические уравнения. Моделирование представляет собой виртуальную среду, в которой пользователи могут взаимодействовать с атомами и молекулами, чтобы узнать, как сбалансировать химические уравнения. В игре пользователям предоставляется химическое уравнение, и им нужно настроить количество атомов каждого элемента в уравнении, чтобы сбалансировать его.
Моделирование позволяет пользователям добавлять или удалять атомы различных элементов, а также обеспечивает обратную связь о том, сбалансировано уравнение или нет. Эта интерактивная игра помогает учащимся понять концепцию балансировки химических уравнений, которая является важной частью химии. Игра позволяет игрокам проверить, является ли уравнение правильным представлением химической реакции, и помогает учащимся понять, как атомы трансформируются во время химической реакции.
Ученики могут поэкспериментировать с различными способами уравновешивания уравнений и сразу же узнать, верны они или нет. Это помогает им быстрее и эффективнее освоить процесс балансировки химических уравнений.
Ссылка на игру
7. Калькулятор балансировки химических уравнений
Это интересный способ научиться балансировать химические уравнения. Игра показывает вам изображение некоторых химических веществ и просит сбалансировать изображение. Думайте об этом как о сборке пазла — вам нужно убедиться, что все подходит правильно.
Вам нужно подсчитать количество каждого химического вещества, а затем сложить числа, чтобы картинка была сбалансированной. Например, если на одной стороне есть два атома кислорода, вам нужно убедиться, что на другой стороне также есть два атома кислорода. Вы вводите числа, которые, по вашему мнению, уравновесят изображение, а затем нажимаете кнопку, чтобы проверить, правы ли вы. Если да, то игра скажет вам, что вы хорошо поработали! Если нет, вы можете попробовать еще раз, пока не получите правильный результат.
Игра отслеживает, сколько картинок вы правильно сбалансировали, чтобы вы могли видеть, насколько хорошо у вас получается. Имейте в виду, что при вводе формулы нет необходимости добавлять пробел. В целом, это похоже на головоломку, которую нужно решить, и вы увидите, как многому вы научились по ходу дела!
Game Link
Овладение искусством балансировки химических уравнений: стратегии и советы для достижения успеха
Балансировку химических уравнений можно упростить, следуя нескольким ключевым стратегиям:
Начните с написания неуравновешенного уравнения с реагентами слева и продуктами справа.
Подсчитайте количество атомов каждого элемента в обеих частях уравнения и составьте список этих чисел.
Определите несбалансированные элементы и сначала выберите один из них для балансировки.
Используйте коэффициенты (числа, расположенные перед химическими формулами), чтобы сбалансировать выбранный элемент в одной части уравнения.
Отрегулируйте коэффициенты других элементов, чтобы сохранить баланс, и повторите процесс для всех оставшихся несбалансированных элементов.
Наконец, убедитесь, что число атомов каждого элемента одинаково в обеих частях уравнения.
Несколько советов по облегчению процесса балансировки:
Начните с самых сложных молекул или молекул с наибольшим количеством атомов.
Используйте дроби, если необходимо сбалансировать уравнение. Например, вместо O можно использовать 1/2 O2, а вместо H можно использовать 2/3 h3.
Помните, что балансировка уравнения требует корректировки как коэффициентов, так и нижних индексов химических формул.
Еще раз проверьте свою работу, чтобы убедиться, что уравнение полностью сбалансировано, а коэффициенты представлены в простейшей форме.
Следуя этим стратегиям и советам, составление химических уравнений может стать веселой и интересной задачей, которая поможет углубить ваше понимание химии. В то же время химия также может быть облегчена для детей с нарушениями обучаемости, такими как дискалькулия.
Заключение
В заключение, использование балансирующих онлайн-игр с уравнениями в качестве метода обучения для учащихся может быть эффективным и увлекательным способом помочь учащимся изучить химию и улучшить свое понимание предмета. Эти игры предоставляют учащимся интерактивный и визуальный опыт обучения, который может помочь привлечь их внимание и сделать процесс обучения более приятным.
С помощью множества доступных научных и математических игр учащиеся могут преодолеть неправильные представления и глубже понять концепции. Учителя могут использовать эти игры в качестве дополнения к традиционным занятиям в классе, а учащиеся могут использовать их для повторения и закрепления того, что они узнали.
Калькулятор уравнений баланса | Balance Chemical Equation
Удобный онлайн-калькулятор уравнений балансировки поможет вам легко сбалансировать химические уравнения. Просто введите свое химическое уравнение в поле ввода и нажмите кнопку расчета, чтобы проверить результат за считанные секунды с подробной работой.
Введите реагенты
Введите продукты
Калькулятор уравнений балансировки: Free Balance Equations Tool — это замечательный инструмент, который поможет вам сбалансировать любое химическое уравнение. Кроме того, в нем описаны простые шаги для балансировки химического уравнения, а также определение уравнения балансировки. Баланс химического уравнения важен в некоторых ситуациях. Он добавит коэффициенты к реагентам и продуктам, чтобы сбалансировать уравнение. Выполняя этот процесс, мы можем сказать, что количество молекул с правой стороны равно количеству молекул с левой стороны.
Здесь мы даем 5 простых шагов, чтобы сбалансировать любое химическое уравнение. Следуйте этим рекомендациям и сбалансируйте свое уравнение легко и быстро.
Подсчитайте количество атомов с каждой стороны уравнения
Проверьте коэффициенты атомов и добавьте любое число n в качестве коэффициента для требуемых атомов, чтобы сбалансировать уравнение.
Разрешается только ставить коэффициент перед реагентом или продуктом, но не разрешается изменять нижние индексы элементов.
После добавления коэффициентов один раз проверьте, чтобы количество атомов в левой части было равно количеству атомов в правой части.
Измените коэффициенты для необходимых атомов, чтобы сбалансировать уравнение.
Когда вы имеете дело с многоатомными ионами, рассматривайте каждый многоатомный ион как один элемент.
Добавьте коэффициенты к этим пунктам, чтобы получить сбалансированное уравнение.
Иногда подсчитайте все атомы в химическом уравнении, даже если оно содержит многоатомные ионы, и добавьте к нему коэффициенты, чтобы получить сбалансированное уравнение.
. Написать по номеру . Левая сторона имеет Ca=1, Cl= 2, Ag= 1, N= 1, O= 3
В правой части Ca=1, Cl=1, Ag=1, N=2, O=6
Атомы азота, хлора и кислорода не сбалансированы.
Добавьте 2 слитка AgNO 3 и AgCl.
Левая сторона содержит Ca=1, Cl= 2, Ag= 1*2 = 2, N= 1*2 = 2, O= 3*2 = 6
Правая сторона содержит Ca= 1, Cl= 1 *2 = 2, Ag= 1*2 = 2, N= 2, O= 6
Наконец, уравнение сбалансировано.
CaCl 2 +2 AgNO 3 → Ca(NO 3 ) 2 + 2AgCl
Onlinecalculator.guru — лучшее место, где вы можете найти калькуляторы различных математических, физических и химических концепций. Взгляните на них и используйте, когда это необходимо, чтобы получить мгновенные результаты.
сообщите об этом объявлении
1. Каковы правила балансировки химического уравнения?
Примените закон сохранения массы, чтобы получить одинаковое количество атомов с каждой стороны химического уравнения.
Начните с балансировки элемента, доступного только в одном реагенте и продукте.
После завершения балансировки одного элемента продолжайте, пока все элементы уравнения не будут сбалансированы.
Сбалансируйте химическое уравнение, поставив перед ним коэффициенты. Вы не должны добавлять индексы, потому что это изменит реакцию.
2. Вы складываете или умножаете балансирующие уравнения?
Важный момент, который следует помнить при балансировке уравнения: никогда не меняйте его индексы, добавляйте к ним коэффициенты. Всегда умножайте коэффициенты атомов на его нижний индекс, чтобы определить количество элементов в реакции.
3. Как сбалансировать длинное химическое уравнение с помощью алгебраического метода?
Сначала ко всем реагентам и продуктам добавляем переменные a,b,c,d и т.д.
Обратите внимание на количество элементов справа и слева.
Приравняйте любой элемент количеству молекул с обеих сторон.
Простое руководство для тех, кому нужно вспомнить школьную программу или помочь ребёнку.
Какие бывают дроби
Дробь — это число, которое состоит из одной или из нескольких равных частей единицы. Говоря упрощённо, это число обозначает часть чего‑либо, например один кусок торта, или целое с несколькими дополнительными частями, например один целый торт и ещё три куска другого.
Обыкновенные дроби состоят из числителя (вверху) и знаменателя (внизу), разделённых горизонтальной или косой чертой. Знаменатель отражает то, на сколько частей можно разделить наш условный торт, а числитель — сколько из них в наличии: 1/2, 3/4, 9/10.
Обыкновенные дроби бывают правильные и неправильные. У правильных числитель меньше знаменателя (5/8, 7/15), а у неправильных наоборот — больше (8/5, 15/7). Из неправильной дроби можно выделить целую и дробную части: 13/5, 21/7. Получившееся число будет называться смешанной дробью.
Бывают ещё десятичные дроби. У них в знаменателе стоит степень числа 10, и они записываются по‑другому — через запятую: 0,5, 0,98. Хотя десятичные дроби можно представить и в виде обыкновенных: 5/10, 98/100.
Как складывать дроби
Обыкновенные с одинаковыми знаменателями
Чтобы сложить дроби с одинаковыми знаменателями, просто суммируйте их числители, а знаменатели оставьте без изменений. Например: 1/5 + 2/5 = 3/5; 9/6 + 10/6 = 19/6 = 31/6.
Обыкновенные с разными знаменателями
Сначала нужно привести дроби к общему знаменателю. Для этого найдите наименьшее число, которое без остатка делится на оба ваших знаменателя. Например, для дробей 5/6 и 4/9 это число 18.
Затем разделите его на ваши знаменатели — и вы получите так называемый дополнительный множитель (18 : 6 = 3, 18 : 9 = 2). Это число, на которое нужно умножить обе части дроби, чтобы привести её к новому знаменателю. То есть: 5 x 3/6 x 3 + 4 x 2/9 x 2 = 15/18 + 8/18.
Остаётся только повторить процесс из предыдущего пункта, сложив числители. В нашем примере получится 23/18, или 15/18, если выделить целую часть.
Смешанные дроби
Складывать такие дроби можно несколькими способами. Самый простой — суммировать целые и дробные части отдельно. Например, вам нужно сосчитать, сколько будет 31/5 + 42/3. Сначала складываем 3 + 4 и получаем 7. Потом переходим к дробным частям: 1/5 + 2/3 = 1 x 3/5 x 3 + 2 x 5/3 x 5 = 3/15 + 10/15 = 13/15. А вместе — 713/15.
Если при сложении дробных частей получается неправильная дробь, из неё тоже нужно выделить целое и добавить к полученной ранее целой части.
Десятичные дроби
Первым делом нужно уравнять количество цифр после запятой. Например, вы хотите сложить числа 33,142 и 5,6. Добавьте два нуля ко второй дроби — 5,600. Теперь сложите между собой числа до запятой (33 + 5) и после (142 + 600). Получится 38,742.
Если вы ещё не очень хорошо освоили работу с десятичными дробями, суммируйте их столбиком, как обычные числа. Следите за тем, чтобы запятая была под запятой. Такой метод сложения облегчит вам подсчёты в том случае, когда после запятой появляется «лишняя» цифра.
Например, нужно найти сумму чисел 1,742 и 5,6. Вы уже знаете, что 1 + 5 = 6, а 742 + 600 = 1 342, но в столбике вы сразу увидите, что единицу из 1 342 нужно перенести, добавить к целой части. В итоге получится 7,342.
Читайте также 🧐
7 причин полюбить математику
11 книг, которые прокачают математическое мышление
10 увлекательных задач от советского математика
Обыкновенные дроби
Урок 3.
Подготовка к ОГЭ по математике 9 класс
Очень часто мы решаем задачи, в которых нельзя выполнить целочисленное деление. В таких ситуациях можно частное записать в виде дроби. На этом уроке мы вспомним, что обыкновенные дроби бывают правильными и неправильными, а также повторим основное свойство дроби, которое позволяет сокращать ее и приводить к новому знаменателю. Вспомним, как неправильную дробь можно представить в виде смешанной или в виде целого числа, а также как смешанную дробь можно представить в виде неправильной. Освежив в памяти правила выполнения действий над обыкновенными дробями, мы применим их при вычислении значений выражений.
Конспект урока «Обыкновенные дроби»
Вопросы
занятия:
· повторить понятие «обыкновенная дробь», виды
обыкновенных дробей;
· повторить основное свойство дроби;
· вспомнить, как неправильную дробь можно
представить в виде смешанной или целого числа, а также как смешанную дробь
можно представить в виде неправильной;
· повторить порядок выполнения действий над обыкновенными
дробями.
Материал урока
Мы
ранее рассматривали случаи, когда нельзя выполнить целочисленное деление. В
таких ситуациях можно частное записать в виде дроби. Делимое тогда называют
числителем, а делитель — знаменателем. Отделяет их друг от друга черта дроби.
Как
вам известно выделяют правильные и неправильные обыкновенные дроби. Напомним их
отличия.
Напомним
основное свойство дроби.
Числитель
и знаменатель дроби можно умножать и делить на одно и тоже число, при этом
значение дроби останется тем же.
Определение.
Умножение
числителя и знаменателя на некоторое число называют приведением к новому
знаменателю. Это позволяет приводить дроби к общему знаменателю.
Обычно
дроби приводят к наименьшему общему знаменателю. Он равен наименьшему общему
кратному знаменателей данных дробей.
Определение.
Процесс
деления числителя и знаменателя на некоторое число мы привыкли называть сокращением.
Обычно
сократимую дробь сокращают на наибольший общий делитель числителя и
знаменателя. Тем самым в итоге получают несократимую дробь. И к такому виду
принято приводить все дроби, полученные в результате вычислений, прежде чем
записать ответ. Дробь является несократимой, если числитель и знаменатель
являются взаимно простыми числами.
Пример.
А
теперь сократим дроби.
Пример.
Вам
хорошо известно, что у любой неправильной дроби можно выделить целую часть.
Напомним,
как это можно сделать.
Можно
разделить числитель на знаменатель с остатком.
Частное
будет целой частью, остаток — числителем дробной части, а исходный знаменатель
— знаменателем дробной части.
Так
из неправильной дроби мы получим смешанную.
Пример.
Далее
вспомним правила сравнения обыкновенных дробей.
Если
же у дробей разные числители и разные знаменатели, то пользуясь основным
свойством дроби их можно привести или к равным знаменателям, или к равным
числителям.
В
работе с дробями нужно уметь не только выделять целую часть у неправильных
дробей, а ещё и представлять смешанные дроби в виде неправильных. Напомним, как
это можно сделать.
Для
этого в числитель записывают произведение целой части и знаменателя,
увеличенного на числитель исходной дроби. Ну, а знаменатель оставляют тем же.
Пример.
Далее
подробнее поговорим о выполнении арифметических действий с дробями.
Складывая
дроби с одинаковыми знаменателями, в числитель записываем сумму числителей, а
знаменатель оставляем тем же.
Если
же нужно сложить дроби с разными знаменателями, предварительно их нужно
привести к общему и сложить полученные дроби.
Пример.
Вычитание
дробей с одинаковыми знаменателями проводят по аналогичному правилу. В
числитель записывают разность числителей, а знаменатель оставляют тем же.
Для
вычитания дробей с разными знаменателями, их сначала нужно привести к общему
знаменателю, а затем вычислить разность полученных дробей.
Пример.
Теперь
поговорим об умножении дробей.
Чтобы
умножить дробь на число нужно только числитель умножить на это число, а
знаменатель оставить тем же.
Произведением
двух дробей является дробь, у которой числитель равен произведению числителей
исходных дробей, а знаменатель — произведению знаменателей исходных дробей.
К
тому же вам известно, что, если среди дробей множителей есть смешанные дроби,
то их нужно предварительно представить в виде неправильных.
Пример.
А
теперь самое время вспомнить понятие взаимно обратных чисел.
Определение.
Взаимно
обратными называют 2 числа, произведение которых равно единице.
Например,
Вернёмся
к действиям с дробями и рассмотрим последнее — деление дробей.
Прежде
чем приступить к делению, смешанные дроби так же нужно представлять в виде
неправильных, и далее пользоваться таким правилом.
Знак
деления нужно заменить умножением и дробь-делитель заменить обратной ей дробью.
А далее следовать по правилу умножения дробей.
Пример.
Пример.
Итоги урока
Подводя итоги урока, вспомним, какие вопросы мы на нём
осветили.
Мы вспомнили, что обыкновенные дроби бывают
правильными и неправильными, а также повторили основное свойство дроби, которое
позволяет сокращать дроби и приводить их к новому знаменателю.
Вспомнили, как неправильную дробь можно представить в
виде смешанной или в виде целого числа, а также как смешанную дробь можно
представить в виде неправильной.
И, освежив в памяти правила выполнения действий над
обыкновенными дробями, мы применили их при вычислении значений выражений.
Предыдущий урок 2
Целые числа
Следующий урок 4
Десятичные дроби
Получите полный комплект видеоуроков, тестов и презентаций
Подготовка к ОГЭ по математике 9 класс
Чтобы добавить комментарий зарегистрируйтесь или войдите на сайт
Повторяющееся десятичное число в дробь — этапы преобразования, хитрости, примеры
Очень легко преобразовать завершающее десятичное число в дробное, но как преобразовать повторяющееся десятичное число в дробное? Повторяющиеся десятичные числа — это десятичные числа, которые не заканчиваются после конечного числа цифр, и в этих числах одна или несколько цифр повторяются снова и снова. Например, 34,56565656… Повторяющееся преобразование десятичной дроби в дробную можно выполнить, выполнив несколько простых шагов, приведенных ниже.
1.
Как преобразовать повторяющуюся десятичную дробь в дробь?
2.
Трюк с повторением десятичной дроби
3.
Повторение таблицы десятичной дроби
4.
Часто задаваемые вопросы о повторении десятичной дроби
Как преобразовать повторяющуюся десятичную дробь в дробь?
Повторяющиеся или повторяющиеся десятичные числа — это такие десятичные расширения, которые не заканчиваются или не заканчиваются после определенного количества цифр. Такие числа имеют бесконечное количество знаков после запятой. И в этих цифрах есть повторяющийся узор.
Как правило, десятичные числа можно преобразовать в дроби путем деления числа в степени 10, равной количеству знаков после запятой. Например, 1,5 = 15/10 = 3/2. Но с повторяющимися десятичными знаками невозможно подсчитать количество знаков после запятой, так как оно бесконечно. Итак, есть несколько конкретных шагов, которые необходимо выполнить, чтобы преобразовать повторяющуюся десятичную дробь в дробь. Повторяющиеся шаги преобразования десятичной дроби в дробную часть приведены ниже:
Шаг 1: Найдите повторяющиеся цифры в заданном десятичном числе.
Шаг 2: Приравняйте десятичное число к x или любой другой переменной.
Шаг 3: Поместите повторяющиеся цифры слева от десятичной точки, умножив уравнение, полученное на шаге 2, на степень 10, равную количеству повторяющихся цифр. Таким образом, вы получите еще одно уравнение.
Шаг 4: Вычтите уравнение, полученное на шаге 2, из уравнения, полученного на шаге 3.
Шаг 5: Упростите, чтобы получить ответ.
Давайте рассмотрим пример, чтобы лучше понять преобразование повторяющейся десятичной дроби в дробь. Преобразуйте 0,77777… в дробь.
Шаг 1: Мы можем заметить, что 7 повторяется в заданном десятичном числе.
Шаг 2: Пусть x = 0,7777…
Шаг 3: Имеется только 1 повторяющаяся цифра, поэтому умножьте это уравнение на 10. Получаем 10x = 7,7777…
Шаг 4: Вычтем x = 0,7777. .. из 10х = 7,7777… Получим 9х = 7.
Шаг 5: х = 7/9. Следовательно, 0,7777… = 7/9.
Когда мы рассматриваем преобразование повторяющихся десятичных чисел в дроби, возникают два типа повторяющихся чисел. Они приведены ниже:
Числа, содержащие только повторяющиеся цифры, например, 0,222…, 0,999…, 0,787878… и т. д.
Многозначные числа, например, 2,7646464…, 98,735735735… и т. д.
Мы уже обсуждали, как преобразовать первый тип повторяющихся десятичных дробей в дроби. Теперь давайте преобразуем многозначное повторяющееся десятичное число до дроби . Преобразуйте 3,989898… в дробь.
Шаг 1: Повторяющиеся цифры в заданном десятичном числе равны 98.
Шаг 2: Пусть x = 3,989898…
Шаг 3: Поскольку есть две повторяющиеся цифры, умножьте приведенное выше уравнение на 100. Мы получим , 100x = 398,989898…
Шаг 4: Вычтем x = 3,989898… из 100x = 398,989898… Получим 99x = 395.
Шаг 5: x = 395/99. Следовательно, 3,989898… = 395/99.
Возьмем еще один пример, в котором нам нужно составить три уравнения, чтобы преобразовать число в дробь. Преобразуйте 2,7646464… в дробь.
Сначала приравняем его к переменной x. Итак, x = 2,7646464… Теперь повторяющиеся цифры равны 64. Итак, давайте умножим это уравнение на 10 так, чтобы у нас были повторяющиеся цифры после запятой. Отсюда следует, что 10x = 27,6464…
10x = 27,6464… (уравнение 1)
Теперь давайте умножим x = 2,7646464… на 1000, чтобы у нас была десятичная точка справа от повторяющихся цифр. .
1000x = 2764,6464… (уравнение 2)
Вычтем уравнение 1 из уравнения 2, получим,
1000x — 10x = (2764,6464. ..) — (27,6464…)
2 9930x =
х = 2737/990
Следовательно, 2,7646464… = 2737/990.
Повторение трюка с десятичной дробью
Чтобы преобразовать повторяющуюся десятичную дробь в форму дроби, достаточно просто записать повторяющиеся цифры в числителе одного и того же числа девяток. Некоторые примеры повторения десятичной дроби приведены ниже:
0,444… = 4/9 (поскольку есть только 1 повторяющаяся цифра 4, поэтому в знаменателе будет только одна 9)
0,787878… = 78/99 (поскольку есть 2 повторяющиеся цифры 7 и 8, значит дважды 9, т.е. 99 будет в знаменателе)
0,999… = 9/9 = 1
Обратите внимание, что этот трюк применим только к повторяющимся десятичным дробям только с повторяющимися цифрами. В данном числе не должно быть других цифр. В этом случае нам нужно использовать повторяющиеся шаги преобразования десятичных дробей в дробные, описанные выше.
Повторение таблицы десятичных дробей
Таблица повторяющихся десятичных дробей поможет вам получить дробные значения некоторых часто используемых повторяющихся десятичных дробей. Вы можете попытаться преобразовать повторяющиеся десятичные дроби (написанные слева) в дроби, а затем использовать приведенную ниже таблицу, чтобы проверить свои ответы.
Статьи по теме
Проверьте эти интересные статьи, связанные с концепцией повторения десятичной дроби в математике.
Преобразование десятичной дроби в дробную
Повторяющееся десятичное число
Что такое 0,6, повторяющееся как дробь?
Что такое 0,4, повторяющееся как дробь?
Что такое 0,3, повторяющееся как дробь?
Часто задаваемые вопросы о повторении десятичной дроби
Что такое преобразование повторяющейся десятичной дроби в дробь?
Преобразование повторяющегося десятичного числа в дробь означает нахождение дробного эквивалента повторяющегося или повторяющегося десятичного представления. Мы можем легко преобразовать дробь в повторяющуюся десятичную, разделив числитель на знаменатель. Но чтобы преобразовать повторяющуюся десятичную дробь в дробь, нам нужна определенная процедура.
Как преобразовать повторяющуюся десятичную дробь в дробь?
Чтобы заменить повторяющуюся десятичную дробь, выполните шаги, указанные ниже:
Определите набор повторяющихся цифр в заданном десятичном числе.
Составьте два уравнения: одно с десятичной точкой слева от повторяющихся цифр, а другое с десятичной точкой справа от первого набора повторяющихся цифр.
Для этого приравняйте данное число к любой переменной и умножьте уравнение на подходящую степень 10.
После этого мы вычитаем меньшее уравнение из большего, чтобы получить дробный эквивалент заданной повторяющейся десятичной дроби.
Как преобразовать непрерывающееся повторяющееся десятичное число в дробь?
Неконечные повторяющиеся десятичные дроби и повторяющиеся десятичные дроби — это одно и то же, поскольку все повторяющиеся десятичные дроби также являются неконечными. Чтобы преобразовать непрерывающуюся повторяющуюся десятичную дробь в дробь, нам нужно выполнить те же шаги, что и при преобразовании повторяющейся десятичной дроби в дробь.
Что такое 0,22… Повторение десятичной дроби?
В заданном числе 0,22… есть одна повторяющаяся цифра, равная 2. Таким образом, используя трюк с повторяющейся десятичной дробью, мы знаем, что 0,22… можно записать как 2/9.
Как преобразовать повторяющуюся десятичную дробь в дробь?
Ниже приведены шаги для преобразования повторяющейся десятичной дроби в дробь:
Определите повторяющиеся цифры в заданном числе.
Приравнять число к любой переменной.
Умножьте обе части уравнения на степень 10 так, чтобы десятичная точка находилась справа от повторяющейся цифры.
Если нужно, найдите еще одно уравнение, в котором все повторяющиеся цифры будут справа от запятой.
Вычтите меньшее уравнение из большего уравнения.
Это метод преобразования повторяющейся десятичной дроби в дробь.
Преобразование повторяющихся десятичных дробей в дроби
При преобразовании повторяющихся десятичных дробей в дроби просто тщательно выполните следующие пять шагов.
Шаг 1:
Пусть x равно повторяющейся десятичной дроби, которую вы пытаетесь преобразовать в дробь.
Шаг 2:
Просмотрите повторяющиеся десятичные числа, чтобы найти повторяющиеся цифры.
Шаг 3:
Поместите повторяющиеся цифры слева от десятичной точки.
Шаг 4:
Поместите повторяющиеся цифры справа от десятичной точки.
Шаг 5:
Используя два уравнения, которые вы нашли в шагах 3 и 4, вычтите левые части двух уравнений. Затем вычтите правые части двух уравнений
При вычитании убедитесь, что разница положительна для обеих сторон.
Теперь давайте потренируемся преобразовывать повторяющиеся десятичные дроби в дроби на двух хороших примерах.
Пример №1:
Какое рациональное число или дробь равно 0,55555555555
Шаг 1:
x = 0,55555555555
Шаг 2:
После обследования повторяющаяся цифра составляет 5.
Шаг 3:
. десятичной точки, вам нужно переместить десятичную точку на 1 разряд вправо.
Технически перемещение десятичной точки на одну позицию вправо осуществляется путем умножения десятичного числа на 10.
Когда вы умножаете одну сторону на число, вы должны умножить другую сторону на то же число, чтобы сохранить уравнение сбалансированный.
Таким образом, 10x = 5,555555555
Шаг 4:
Поместите повторяющиеся цифры справа от десятичной точки.
Еще раз посмотрите на уравнение в шаге 1. В этом примере повторяющаяся цифра уже справа, поэтому больше ничего не нужно делать.
X = 0,55555555555
Шаг 5:
Ваши два уравнения:
10x = 5,555555555
x = 0,5555555555
10x -x = 5,55555555555555
10x -x = 5,5555555555555
.0003
9x = 5
Разделение обеих сторон на 9.
x = 5/9
Пример № 2:
Какое рациональное число или дробь равна 1,04242424242
Шаг 1:
. 1.04242424242
Шаг 2:
После проверки повторяющаяся цифра будет 42.
Шаг 3:
Чтобы поместить повторяющуюся цифру ( 42 ) влево от десятичной запятой укажите на 3 позиции вправо.
Опять же, перемещение десятичной точки на три знака вправо осуществляется путем умножения десятичного числа на 1000.
Когда вы умножаете одну часть на число, вы должны умножить другую часть на то же число, чтобы уравнение оставалось сбалансированным
Таким образом, 1000x = 1042,42424242
Шаг 4:
Поместите повторяющиеся цифры справа от десятичной точки.
В этом примере повторяющаяся цифра не находится сразу справа от десятичной точки.
Посмотрите на уравнение в шаге 1 еще раз, и вы увидите, что между повторяющейся цифрой и десятичной точкой стоит ноль.
Для этого нужно переместить запятую на 1 разряд вправо.
Это выполняется путем умножения обеих сторон на 10.
Чтобы усвоить этот урок о преобразовании повторяющихся десятичных дробей в дроби, вам необходимо внимательно изучить два приведенных выше примера и попрактиковаться на других примерах.
Распространенные ошибки, которых следует избегать при преобразовании повторяющихся десятичных знаков в дроби
Забыть поставить десятичную точку непосредственно перед повторяющейся цифрой (цифрами).
При вычитании двух уравнений забывают вычесть меньшее из большего.
Недостаточное отслеживание количества знаков, на которые была перемещена десятичная точка.
Думая, что повторяющаяся цифра (цифры) равна 8, если десятичное число равно 0,08
Квадратичная формула: простые шаги
26, 23 января 11:44
Узнайте о квадратной формуле, дискриминанте, важных определениях, связанных с формулой, и приложениях.
Касательная плоскость и нормаль к явно заданной поверхности.
Касательной плоскостью к поверхности в ее точке $M_0$ (точка касания) называется плоскость, содержащая в себе все касательные к кривым, проведенным на поверхности через эту точку.
Нормалью к поверхности называется прямая, перпендикулярная к касательной плоскости и проходящая через точку касания.
Если уравнение поверхности имеет вид $$F(x,y,z)=0,$$ то уравнение касательной плоскости в точке $M_0(x_0, y_0, z_0)$ есть $$F_x'(x_0, y_0, z_0)(x-x_0)+F_y'(x_0, y_0, z_0)(y-y_0)+F_z'(x_0, y_0, z_0)(z-z_0)=0.$$
В случае задания поверхности в явной форме $$z=f(x, y)$$ уравнение касательной плоскости в точке $M_0(x_0, y_0, z_0)$ имеет вид $$z-z_0=f_x'(x_0, y_0)(x-x_0)+f_y'(x_0, y_0)(y-y_0),$$ а уравнение нормали $$\frac{x-x_0}{f_x'(x_0, y_0)}=\frac{y-y_0}{f_y'(x_0, y_0)}=\frac{z-z_0}{-1}. $$
Примеры:
7.229. а) Найти уравнения касательной плоскости и нормали к поверхности $z=\sin x\cos y$ в точке $(\pi/4, \pi/4, \pi/4).$
Решение.
Для поверхности $$z=f(x, y)$$ уравнение касательной плоскости в точке $M_0(x_0, y_0, z_0)$ имеет вид $$z-z_0=f_x'(x_0, y_0)(x-x_0)+f_y'(x_0, y_0)(y-y_0),$$ а уравнение нормали $$\frac{x-x_0}{f_x'(x_0, y_0)}=\frac{y-y_0}{f_y'(x_0, y_0)}=\frac{z-z_0}{-1}.$$
Таким образом, уравнение касательной плоскости: $$z-\frac{\pi}{4}=\frac{1}{2}(x-\frac{\pi}{4})-\frac{1}{2}(y-\frac{\pi}{4})\Rightarrow$$ $$\frac{1}{2}x -\frac{1}{2}y-z+\frac{\pi}{4}=0. 2-2x+6y=4$ найти уравнение нормали, параллельной прямой $\frac{x+2}{1}=\frac{y}{3}=\frac{z+1}{4}.$
Дифференциальное и интегральное исчисления для втузов, т.1
Пискунов Н. С. Дифференциальное и интегральное исчисления для втузов, т.1: Учебное пособие для втузов.— 13-е изд.— М.: Наука. Главная редакция физико-математической литературы, 1985. — 432 с.
Хорошо известное учебное пособие по математике для втузов с достаточно широкой математической подготовкой.
Первый том включает разделы: введение в анализ, дифференциальное исчисление (функций одной и нескольких переменных), неопределенный и определенный интегралы.
Настоящее издание не отличается от предыдущего (1978 г.).
Для студентов высших технических учебных заведений.
Оглавление
ПРЕДИСЛОВИЕ К ДЕВЯТОМУ ИЗДАНИЮ ПРЕДИСЛОВИЕ К ПЯТОМУ ИЗДАНИЮ ГЛАВА I. ЧИСЛО. ПЕРЕМЕННАЯ. ФУНКЦИЯ § 1. Действительные числа. § 2. Абсолютная величина действительного числа § 3. Переменные и постоянные величины § 4. Область изменения переменной величины § 5. Упорядоченная переменная величина. Возрастающая и убывающая переменные величины Ограниченная переменная величина § 6. Функция § 7. Способы задания функции § 8. Основные элементарные функции. Элементарные функции § 9. Алгебраические функции § 10. Полярная система координат Упражнения к главе I ГЛАВА II. ПРЕДЕЛ. НЕПРЕРЫВНОСТЬ ФУНКЦИЙ § 1. Предел переменной величины. Бесконечно большая переменная величина § 2. Предел функции § 3. Функция, стремящаяся к бесконечности. Ограниченные функции § 4. Бесконечно малые и их основные свойства § 5. Основные теоремы о пределах § 6. Предел функции (sin x)/x при x->0 § 7. Число e § 8. Натуральные логарифмы § 9. Непрерывность функций § 10. Некоторые свойства непрерывных функций § 11. n при n целом и положительном § 6. Производные от функций y = sinx; y = cosx § 7. Производные постоянной, произведения постоянной на функцию, суммы, произведения, частного § 8. Производная логарифмической функции § 9. Производная от сложной функции § 10. Производные функций y = tgx, y = ctgx, y = ln|x| § 11. Неявная функция и ее дифференцирование § 12. Производные степенной функции при любом действительном показателе, показательной функции, сложной показательной функции § 13. Обратная функция и ее дифференцирование § 14. Обратные тригонометрические функции и их дифференцирование § 15. Таблица основных формул дифференцирования § 16. Параметрическое задание функции § 17. Уравнения некоторых кривых в параметрической форме § 18. Производная функции, заданной параметрически § 19. Гиперболические функции § 20. Дифференциал § 21. Геометрическое значение дифференциала Рассмотрим функцию § 22. Производные различных порядков § 23. x, sin x, cos x Упражнения к главе IV ГЛАВА V. ИССЛЕДОВАНИЕ ПОВЕДЕНИЯ ФУНКЦИЙ § 2. Возрастание и убывание функции § 3. Максимум и минимум функций § 4. Схема исследования дифференцируемой функции на максимум и минимум с помощью первой производной § 5. Исследование функции на максимум и минимум с помощью второй производной § 6. Наибольшее и наименьшее значения функции на отрезке § 7. Применение теории максимума и минимума функций к решению задач § 8. Исследование функции на максимум и минимум с помощью формулы Тейлора § 9. Выпуклость и вогнутость кривой. Точки перегиба § 10. Асимптоты § 11. Общий план исследования функций и построения графиков § 12. Исследование кривых, заданных параметрически Упражнения к главе V ГЛАВА VI. КРИВИЗНА КРИВОЙ § 1. Длина дуги и ее производная § 2. Кривизна § 3. Вычисление кривизны § 4. Вычисление кривизны линии, заданной параметрически § 5. Вычисление кривизны линии, заданной уравнением в полярных координатах § 6. Радиус и круг кривизны. Центр кривизны. Эволюта и эвольвента § 7. Свойства эволюты § 8. Приближенное вычисление действительных корней уравнения Упражнения к главе VI ГЛАВА VII. КОМПЛЕКСНЫЕ ЧИСЛА, МНОГОЧЛЕНЫ § 1. Комплексные числа. Исходные определения § 2. Основные действия над комплексными числами § 3. Возведение комплексного числа в степень и извлечение корня из комплексного числа § 4. Показательная функция с комплексным показателем и ее свойства § 5. Формула Эйлера. Показательная форма комплексного числа § 6. Разложение многочлена на множители § 7. О кратных корнях многочлена § 8. Разложение многочлена на множители в случае комплексных корней § 9. Интерполирование. Интерполяционная формула Лагранжа § 10. Интерполяционная формула Ньютона § 11. Численное дифференцирование § 12. О наилучшем приближении функций многочленами. Теория Чебышева Упражнения к главе VII ГЛАВА VIII. ФУНКЦИИ НЕСКОЛЬКИХ ПЕРЕМЕННЫХ § 1. Определение функции нескольких переменных § 2. Геометрическое изображение функции двух переменных § 3. Частное и полное приращение функции § 4. Непрерывность функции нескольких переменных § 5. Частные производные функции нескольких переменных § 6. Геометрическая интерпретация частных производных функции двух переменных § 7. Полное приращение и полный дифференциал § 8. Применение полного дифференциала в приближенных вычислениях § 9. Приложение дифференциала к оценке погрешности при вычислениях § 10. Производная сложной функции. Полная производная. Полный дифференциал сложной функции § 11. Производная от функции, заданной неявно § 12. Частные производные различных порядков § 13. Поверхности уровня § 14. Производная по направлению § 15. Градиент § 16. Формула Тейлора для функции двух переменных § 17. Максимум и минимум функции нескольких переменных § 18. Максимум и минимум функции нескольких переменных, связанных данными уравнениями (условные максимумы и минимумы) § 19. Получение функции на основании экспериментальных данных по методу наименьших квадратов § 20. Особые точки кривой Упражнения к главе VIII ГЛАВА IX. ПРИЛОЖЕНИЯ ДИФФЕРЕНЦИАЛЬНОГО ИСЧИСЛЕНИЯ К ГЕОМЕТРИИ В ПРОСТРАНСТВЕ § 1. Уравнения кривой в пространстве § 2. Предел и производная векторной функции скалярного аргумента. Уравнение касательной к кривой. Уравнение нормальной плоскости § 3. Правила дифференцирования векторов (векторных функций) § 4. Первая и вторая производные вектора по длине дуги. Кривизна кривой. Главная нормаль. Скорость и ускорение точки в криволинейном движении § 5. Соприкасающаяся плоскость. Бинормаль. Кручение. § 6. Касательная плоскость и нормаль к поверхности Упражнения к главе IX ГЛАВА X. НЕОПРЕДЕЛЕННЫЙ ИНТЕГРАЛ § 1. Первообразная и неопределенный интеграл § 2. Таблица интегралов § 3. Некоторые свойства неопределенного интеграла § 4. Интегрирование методом замены переменной или способом подстановки § 5. Интегралы от некоторых функций, содержащих квадратный трехчлен § 6. Интегрирование по частям § 7. Рациональные дроби. Простейшие рациональные дроби и их интегрирование § 8. Разложение рациональной дроби на простейшие § 9. Интегрирование рациональных дробей § 10. Интегралы от иррациональных функций § 11. Интегралы вида … § 12. Интегрирование некоторых классов тригонометрических функций § 13. Интегрирование некоторых иррациональных функций с помощью тригонометрических подстановок § 14. О функциях, интегралы от которых не выражаются через элементарные функции Упражнения к главе X ГЛАВА XI. ОПРЕДЕЛЕННЫЙ ИНТЕГРАЛ § 1. Постановка задачи. Нижняя и верхняя интегральные суммы § 2. Определенный интеграл. Теорема о существовании определенного интеграла § 3. Основные свойства определенного интеграла § 4. Вычисление определенного интеграла. Формула Ньютона — Лейбница § 5. Замена переменной в определенном интеграле § 6. Интегрирование по частям § 7. Несобственные интегралы § 8. Приближенное вычисление определенных интегралов § 9. Формула Чебышева § 10. Интегралы, зависящие от параметра. Гамма-функция § 11. Интегрирование комплексной функции действительной переменной Упражнения кглаве XI ГЛАВА XII. ГЕОМЕТРИЧЕСКИЕ И МЕХАНИЧЕСКИЕ ПРИЛОЖЕНИЯ ОПРЕДЕЛЕННОГО ИНТЕГРАЛА § 1. Вычисление площадей в прямоугольных координатах § 2. Площадь криволинейного сектора в полярных координатах § 3. Длина дуги кривой § 4. Вычисление объема тела по площадям параллельных сечений § 5. Объем тела вращения § 6. Площадь поверхности тела вращения § 7. Вычисление работы с помощью определенного интеграла § 8. Координаты центра масс § 9. Вычисление момента инерции линии, круга и цилиндра с помощью определенного интеграла Упражнения к главе XII
многомерное исчисление — В каких точках поверхности касательная плоскость параллельна плоскости $xy$?
спросил
Изменено
6 лет, 5 месяцев назад
Просмотрено
13 тысяч раз
$\begingroup$
При каких очках. {2}x + 3x$ касательная плоскость параллельна плоскости $xy$? 92=3.$$ Это уравнение гиперболы. В любой точке этой гиперболы касательная плоскость поверхности параллельна $OXY.$
$\endgroup$
$\begingroup$
Подсказка
Плоскость, параллельная плоскости $xy-$, будет иметь уравнение вида
$$z=z_0+0.x+0.y.$$
Ваша очередь решать систему
$$\frac{\partial F}{\partial x}(x_0,y_0)=0$$
$$\frac{\partial F}{\partial y}(x_0,y_0)=0$$
$\endgroup$
1.7: Касательные плоскости и нормальные линии
Последнее обновление
Сохранить как PDF
Идентификатор страницы
602
Ларри Грин
Общественный колледж озера Тахо
Касательные плоскости
Пусть \(z = f(x,y)\) — функция двух переменных. Мы можем определить новую функцию \(F(x,y,z)\) трех переменных, вычитая \(z\). Это имеет условие
\[ F(x,y,z) = 0.\nonumber \]
Теперь рассмотрим любую кривую, заданную параметрически с помощью
\[x = x(t), \;\;\; у = у(t), \;\;\; z = z(t).\nonumber \]
Мы можем написать,
\[F(x(t), y(t), z(t)) = 0.\nonumber \]
Дифференцирование обеих частей по \(t\) и использование цепного правила дает
\[F_x(x, y, z) x’ + F_y(x, y, z) y’ + F_z(x , y, z) z’ = 0\nonumber \]
Обратите внимание, что это скалярное произведение функции градиента и вектора \(\langle x’,y’,z’\rangle \),
В частности, вектор градиента ортогонален касательной к любой кривой на поверхности. Это приводит к:
Определение: Касательная плоскость
Пусть \(F(x,y,z)\) определяет поверхность, дифференцируемую в точке \((x_0,y_0,z_0)\), тогда касательная плоскость к \(F ( x, y , z )\) в точке \(( x_0, y_0, z_0)\) плоскость с вектором нормали
\[ \nabla \, F(x_0,y_0,z_0) \nonumber \]
, проходящая через точку \((x_0,y_0,z_0)\). В частности, уравнение касательной плоскости имеет вид
\[ \nabla \, F(x_0,y_0,z_0) \cdot \langle x — x_0 , y — y_0 , z — z_0 \rangle = 0. \nonumber \]
Пример \(\PageIndex{1}\) 92 — xy — z\nonumber \]
затем
\[\nabla F = \langle 6x — y, -x, -1\rangle . \nonumber \]
Теперь используйте формулу нормали точки для плана
\[\langle 4, -1, -1\rangle \cdot \langle x — 1, y — 2, z — 1\rangle = 0\ nonumber \]
или
\[4(x — 1) — (y — 2) — (z — 1) = 0.\nonumber \]
Наконец, мы получаем
\[ 4x — y — z = 1.\nonumber \]
Нормальные линии
Для данных вектора и точки существует единственная линия, параллельная этому вектору, которая проходит через точку. В контексте поверхностей у нас есть вектор градиента поверхности в данной точке. Это приводит к следующему определению.
Определение: нормальная линия
Пусть \(F(x,y,z)\) определяет поверхность, которая дифференцируема в точке \((x_0,y_0,z_0)\), тогда нормальная линия к \(F(x,y,z)\) at \((x_0,y_0,z_0)\) — линия с вектором нормали
\[ \nabla \, F(x_0,y_0,z_0) . \nonumber \]
, проходящая через точку \((x_0,y_0,z_0)\). В частности уравнение нормальной линии
\[ x(t) = x_0 + F_x(x_0,y_0,z_0) t, \nonumber \]
\[ y(t) = y_0 + F_y(x_0,y_0 ,z_0) t, \nonumber \]
\[ z(t) = z_0 + F_z(x_0,y_0,z_0) t. \nonumber \]
Пример \(\PageIndex{2}\)
Найдите параметрические уравнения для нормальной линии к 92y + 1\rangle = \langle 12, 2, 3\rangle .\nonumber \]
Оба эти вектора будут перпендикулярны касательной к кривой в точке , следовательно, их векторное произведение будет параллельно этой касательной. Мы вычисляем
Эта страница под заголовком 1.7: Касательные плоскости и нормальные линии распространяется по незаявленной лицензии, ее автором, ремиксом и/или куратором был Ларри Грин.